You are on page 1of 106

.

IASBABA’S PRELIMS TEST SERIES (AIPTS) 2024 – TEST 7 (ART & CULTURE)

Q.1) Consider the following statements How many of the above given statements
with reference to ancient scripts of India: is/are correct?

1. The ancient script used in the Indus a) Only One


Valley Civilization, which has not yet b) Only Two
been fully deciphered, is known as c) All Three
Indus Script. d) None of the above
2. The script used for writing the
edicts of Emperor Ashoka, which
helped in deciphering ancient Q.3) With reference to the differences
Indian scripts, is known as Brahmi between Ashokan pillars and Iranian pillars,
Script. consider the following statements:
3. The script used for writing the
1. Ashokan pillars were constructed
ancient language of Gandhara,
using multiple pieces of sandstone
which is a mixture of Prakrit and
joined together with cement
Sanskrit, is known as Kharosthi
whereas Iranian pillars were
Script.
composed of monolithic shafts,
How many of the above given statements meaning they were carved from a
is/are correct? single piece of stone.
2. The Ashokan pillars were erected
a) Only Two
independently whereas Iranian
b) All Three
pillars were affixed to state
c) Only One
buildings.
d) None of the above
Which of the given above statements is/are
correct?
Q.2) Consider the following statements:
a) 1 only
1. The people of Harappan civilization, b) 2 only
regardless of social status, spun c) Both 1 and 2
cotton and wool for their textile d) Neither 1 nor 2
needs.
2. The people of Harappan civilization
showcased a strong appreciation Q.4) Consider the following statements
for fashion, evident through the with respect to Shalabhanjika:
varied hairstyles and beard styles
1. In Jain art, the Shalabhanjika is
they embraced.
portrayed as a yakshi holding a tree.
3. Diverse ornaments such as
2. The motif of the Shalabhanjika was
necklaces, headbands, armlets, and
considered a symbol of good
finger rings were exclusively worn
fortune and was incorporated into
by women only.
the embellishment of the stupa.
.
IASBABA’S PRELIMS TEST SERIES (AIPTS) 2024 – TEST 7 (ART & CULTURE)

Which of the given above statements is/are sculpturesofAmaravathi School


correct? were made using ‘white marble’.
3. Mathura school art of the time was
a) 1 only
influenced by Hinduism, Jainism,
b) 2 only
and Buddhism, representing all
c) Both 1 and 2
three religionswhereasAmaravathi
d) Neither 1 nor 2
school was the predominant
influence on this art style was
Buddhism only.
Q.5) With reference to the Gandhara
School, consider the following statements:

1. It was Influenced and developed by Which of the given above statements is/are
traditions of Greek and Roman correct?
sculptors.
a) Only One
2. The depiction of Buddha showed
b) Only Two
him in a spiritual state with wavy
c) All Three
hair, adorned with fewer
d) None
ornaments.
3. The Buddha is seated in the lotus
position, displaying various mudras,
Q.7) “The mudra incorporates the use of
while his face emanates grace.
both hands. The right hand is positioned at
Which of the given above statements is/are the level of the chest, with the palm facing
correct? outward. The tips of the index finger and
thumb of the right hand form a mystical
a) Only One
circle. The left hand is inwardly turned, and
b) Only Two
the index finger and thumb of the left hand
c) All Three
connect to touch the circle formed by the
d) None
right hand.”

Which of the following mudra is described


Q.6) Consider the following statements in the above paragraph?
with reference to the differences between
a) Dhyana Mudra
Mathura and Amaravathi Schools:
b) Uttarabodhi Mudra
1. Mathura School was influenced by c) Dharmachakra Mudra
Greek whereas Amaravathi School d) Varada Mudra
was developed indigenously and
not influenced by external cultures.
2. The sculptures of Mathura were
made using ‘spotted red sandstone’
whereasthe
.
IASBABA’S PRELIMS TEST SERIES (AIPTS) 2024 – TEST 7 (ART & CULTURE)

Q.8) Consider the following pairs of Which of the given above statements is/are
Ashoka’s Major Pillar edicts with its correct?
content:
a) Only One
(Ashoka’s Major Pillar edicts) b) Only Two
(Content) c) All Three
d) None
1. Major Ashoka's principle of
Pillar edict 1 protection of the
people
Q.10) With reference to the features of
2. Major Dhamma as a minimum
temple architecture during Khajuraho
Pillar edict 2 of sins, virtues,
school, consider the following statements:
compassion,
truthfulness and purity 1. The sculptures predominantly
3. Major Duties of Rajukas portrayed erotic themes and took
Pillar edict 3 inspiration from Vatsyayana's Kama
Sutra.
How many pairs is/are correctly matched? 2. The temples were constructed
a) One pair only using sandstone as the primary
building material.
b) Two pairs only
3. The temples were erected on
c) Three pairs
d) None of the above elevated platforms and exhibited a
blend of both Hindu and Jain
religious influences.
Q.9) With reference to the cultural history Which of the given above statements is/are
of India, consider the following statements: correct?
1. The ascent of the Gupta Empire in a) Only One
the 4th century AD marked a b) Only Two
significant milestone in the history c) All Three
of Hindu temple construction, as it d) None
reached its zenith during this time.
2. Buddhist and Jain art did not
receive significant emphasis during Q.11) With reference to the Ajanta Caves,
the Gupta Age. consider the following statements:
3. In the later phase of Gupta rule,
1. Ajanta is a series of rock-cut caves
worship of three primary deities
near Aurangabad in Maharashtra,
spanned India: Vishnu in the north
situated in the Sahyadri ranges
and central regions, Shiva in the
along the Waghora River.
south, and Shakti in the east and
2. The cave paintings in Ajanta were
along the Malabar Coast.
executed using the fresco painting
.
IASBABA’S PRELIMS TEST SERIES (AIPTS) 2024 – TEST 7 (ART & CULTURE)

technique, exhibiting a remarkable Q.13) With reference to the Architecture


level of naturalism. and Sculpture at Mahabalipuram, consider
3. The paintings within the caves the following statements:
primarily revolve around Jainism,
1. The temples at Mahabalipuram,
depicting scenes from the life of
named after the characters of
Mahavira.
Mahabharata.
4. The Ajanta caves were created with
2. The temples are primarily devoted
the support and patronage of the
to the worship of both Shiva and
Vakataka kings.
Vishnu.
Which of the given above statements is/are 3. At the Ganesha Ratha, there are 18
correct? inscriptions written in Tamil using
both Grantha and Nagari scripts.
a) 1, 2 and 3 only
b) 1, 2 and 4 only Which of the given above statements is/are
c) 2, 3 and 4 only not correct?
d) 1, 3 and 4 only
a) Only One
b) Only Two
c) All Three
Q.12) With reference to the features of
d) None
Nagara style of Architecture, consider the
following statements:

1. Temples typically followed the Q.14) With reference to Bronze Nataraja of


Panchayatana style, which involved Chola Period, consider the following
subsidiary shrines arranged in a statements:
cruciform layout around the main
1. The drum, symbolizing the sound of
shrine. creation, is held in the upper right
2. Generally, water tanks or reservoirs
hand.
were found within the temple 2. The upper left hand grasps the
premises.
eternal fire, representing
3. A covered structure was provided destruction.
over the ambulatory passageway or
3. The Nataraja is encompassed by a
pradakshina path encircling the
radiant nimbus, symbolizing the
inner sanctum.
infinite and unceasing cycles of
Which of the given above statements is/are time.
correct? Which of the given above statements is/are
a) Only One correct?
b) Only Two a) Only One
c) All Three b) Only Two
d) None c) All Three
.
IASBABA’S PRELIMS TEST SERIES (AIPTS) 2024 – TEST 7 (ART & CULTURE)

d) None c) Both 1 and 2


d) Neither 1 nor 2

Q.15) Which of the following statements


is/are correct with respect to Prambanan Q.17) Consider the following pairs with
temple? respect to Indo-Islamic architecture
technique:
1. This renowned Hindu temple is
situated in Cambodia, outside of 1. Charbagh style : Dividing a
India. square block into four identical
2. It holds the prestigious designation adjacent gardens.
of being a UNESCO World Heritage 2. Pietra-dura : Inlaying precious-
site. colored stones and gems into walls
3. The temple was constructed under for decoration.
the patronage of Raja Rajendra 3. Foreshortening technique: Creating
Chola. an optical illusion that made the
inscriptions seems nearer than they
How many of the above given statements
truly were.
is/are correct?

a) Only One How many of the above pair(s) is/are


correctly matched?
b) Only Two
c) All Three a) None of the above
d) None b) One only
c) Two only
d) Three
Q.16) With reference to cultural history of
India, consider the following statements:
Q.18) Which architectural style among the
1. The concept of secular buildings
following is alternatively referred to as the
inside the temple premises was
‘Sharqi style’?
introduced during the period of
Vijayanagara rule. a) Bengal School
2. During Hoysala period Stellate plan b) Jaunpur School
newlybrought to temple c) Malwa School
architecture, which the shrines led d) Bijapur School
out in shape of an intricately
designed star.
Q.19) Consider the following statements:
Which of the given above statements is/are
correct? 1. These structures featured spacious
windows, possibly influenced by
a) 1 only
European styles, and were
b) 2 only
.
IASBABA’S PRELIMS TEST SERIES (AIPTS) 2024 – TEST 7 (ART & CULTURE)

embellished with stylized arches Q.21) With reference to the Mogul’s Art
and pillars. and Architecture, consider the following
2. Artificial reservoirs known as statements:
"baulis" were constructed within
1. Jahangir was responsible for the
the premises for water storage.
construction of Salim Chishti's
3. Locally available materials were
tomb in Fathepur Sikri.
utilized in the construction process.
2. Shah Jahan, the son of Jahangir,
4. Some notable examples of this
was responsible for the
architectural style include the Rani
construction of Jahangir's tomb in
Roopmati Pavilion, Jahaz Mahal,
Lahore.
and Ashrafi Mahal.
3. Aurangzeb commissioned the
Based on the information provided above, construction of Bibi Ka Maqbara in
which architectural school is being alluded Aurangabad as a commemoration
to? to his wife Begum Rabia-ud-
Daurani.
a) Bengal School
b) Jaunpur School How many of the above given statements
c) Malwa School is/are correct?
d) Bijapur School
a) One only
b) Two only
c) Three
Q.20) Which of the following monument d) None of the above
is/are located in Fatehpur Sikri?

1. BulandDarwaza
2. Salim Chishti's tomb Q.22) With reference to the Iberian
3. Panch Mahal architectural style, consider the following
4. Ibadat Khana statements:
5. Hiran Minar 1. The regions explored by the
6. Red fort Portuguese witnessed the
Select the correct code: introduction of the Iberian
architectural style with their arrival.
a) 1, 3, 4, 5 and 6 only 2. This style showcased elaborate and
b) 1, 2, 3, 4 and 5 only theatrical designs, aiming to create
c) 1, 2, 5 and 6 only a striking impression by
d) 1, 3, 4 and 5 only incorporating contrasting colours.
3. St. Paul's Church in Diu stands out
as a prominent example of this
architectural style.
.
.
.
.
IASBABA’S PRELIMS TEST SERIES (AIPTS) 2024 – TEST 7 (ART & CULTURE)

India, primarily in Gujarat and the c) 1 and 3 only


Mewar region of Rajasthan, during d) 1 and 2 only
the 11th to 15th centuries.
2. Initially Buddhist iconography
dominated the paintings, but later Q.36) With reference to the Differences
the Jain School adopted similar between Rajput Style and Mughal Style of
themes. Paintings, consider the following
3. Palm leaf was initially used as the statements:
medium for painting, but later they
1. The Rajput style of painting drew
transitioned to paper.
inspiration from mural and fresco
Which of the given above statements is/are forms, while the Mughal style is
correct? rooted in Persian miniature
painting.
a) 1 and 3 only
2. Rajput painting is devotional or
b) 2 and 3 only religious, while Mughal painting
c) 1 and 2 only
emphasizes the Mughal emperor
d) 1, 2 and 3 and his household. Themes of
pomp, battles, and hunting are also
prevalent in Mughal art.
Q.35) Which of the following statements
3. Rajput painting showcases symbols
is/are correct with respect to Mughal
like trees, camels, and falcons,
Paintings?
while Mughal painting emphasizes
1. The paintings of Akbar's era were individuals or elements like lotus,
characterized by the use of three- peacock, and swan.
dimensional figures and continued
Which of the given above statements is/are
utilization of foreshortening
correct?
techniques.
2. Jahangir created an official artistic a) 1, 2 and 3
studio called Tasvir Khana, where b) 2 and 3 only
artists were employed and given c) 1 and 2 only
the opportunity to develop their d) 1 and 3 only
individual styles.
3. Calligraphy was encouraged and
incorporated into the paintings Q.37) “It is a traditional painting style from
during this period. Odisha that blends classical and folk
elements. Created on treated cloth, the
Select the correct answer by using the
colors are derived from natural sources like
codes below:
burnt coconut shells, hingula, ramaraja,
a) 1, 2 and 3 and lamp black. Outlines are drawn with a
b) 2 and 3 only brush in red or yellow, and colors are filled
in. The paintings have decorative
.
IASBABA’S PRELIMS TEST SERIES (AIPTS) 2024 – TEST 7 (ART & CULTURE)

backgrounds with foliage and flowers, 3. The relief sculptures at Sanchi


intricate frames, and a glossy finish depict aspects of Buddhism but not
achieved by applying a coat of lacquer”. Buddha's image.

Which of the following paintings describes Which of the given above statements is/are
the above paragraph? correct?
a) Kalamkari paintings a) Only One
b) Pattachitra paintings b) Only Two
c) Madhubani paintings c) None
d) Kalighat paintings d) All Three

Q.38) Consider the following pairs Metal Q.40) With reference to the Toy making,
Crafts with states: consider the following pairs:
() ()

Metal Craft State Toys State


1. Bidri Craft Kerala 1. Channapatna Karnataka
2. Aranmula Kannadi Karnataka Toys
3. Chandi Tarkashi Odisha 2. Kondapalli Andhra
Toys Pradesh
How many of the above given pairs is/are 3. Kinnal Toys Kerala
correctly matched?
Which of the above given pairs is/are
a) One pair only correctly matched?
b) Two pairs only a) 1 and 3 only
c) Three pairs b) 2 and 3 only
d) None of the above c) 1 and 2 only
d) 1, 2 and 3 only

Q.39) With reference to the Buddhist


Monuments at Sanchi, consider the Q.41) With reference to the Anatomy of
following statements: Indian music, consider the following pairs:
1. These monuments hold significant (Raga Bhed) (Details)
historical and cultural importance
and Recognized as a UNESCO World 1. Shuddha Raag : This
Heritage site in 1989. is a raga in which if any notes that are
2. Sanchi is recognized as the oldest not present in the original composition
surviving Buddhist sanctuary in the are played, its nature and form change.
world. 2. Chhayalag Raag : In
this raga, the nature and form remain
.
IASBABA’S PRELIMS TEST SERIES (AIPTS) 2024 – TEST 7 (ART & CULTURE)

unchanged even if additional notes not Q.43) Consider the following statements
present in its composition are played. with reference to the Dhrupad Style of
3. Sankeerna Raag : This Music:
raga combines two or more ragas.
1. Dhrupad is one of the oldest and
Which of the given above pairs is/are most prominent forms of Carnatic
correctly matched? classical music.
2. Dhrupad gained significant
a) 1 only
recognition and thrived during the
b) 2 and 3 only
reign of Emperor Akbar.
c) 3 only
3. Dhrupad begins with Alap, a
d) 1, 2 and 3 only
wordless melodic exploration that
sets the raga's mood with a gradual
tempo increase.
Q.42) With reference to the ‘Indian Music’,
consider the following statements: Which of the given above statements is/are
correct?
1. No evidence of musical instruments
has been found at Indus Valley a) 1 and 3 only
Civilization sites. b) 1 and 2 only
2. The Vedic era, contains literary c) 2 and 3 only
mentions of music, including the d) 1, 2 and 3 only
Sama Veda's documentation of the
sequential arrangement of the
seven notes of the Kharaharapriya Q.44) Which of the following statements
raga. about Thumri is correct?
3. Khyal compositions are
1. Thumri is a semi-classical form of
characterized by short songs that
Indian music originating in Uttar
typically range from two to eight
Pradesh.
lines.
2. Thumri compositions are
Which of the given above statements is/are exclusively devotional in nature.
correct? 3. Thumri is exclusively sung in
Sanskrit.
a) 1 and 3 only
4. Thumri is primarily performed by
b) 2 and 3 only
female vocalists.
c) 1 and 2 only
d) 1, 2 and 3 only Select the correct answer by using the code
below:

a) 1 and 3 only
b) 1 and 2 only
c) 1 and 4 only
d) 1, 2 and 4 only
.
IASBABA’S PRELIMS TEST SERIES (AIPTS) 2024 – TEST 7 (ART & CULTURE)

Q.45) Which of the following composers is d) 1, 2, 3, 4, 6 and 7 only


not considered part of the Trinity of
Carnatic Music?
Q.48) Consider the following statements:
a) Thyagarajan
b) Purandaradasa 1. The dance form originated from the
c) Muthuswamy Dikshitar solo performances of temple
d) Syama Sastri dancers or devadasis.
2. E. Krishna Iyer played significant
roles in reviving and popularizing it.
Q.46) Match the following Folk dances with 3. It is often referred to as the 'fire
their respective origin states: dance' due to its resemblance to
the movements of a dancing flame.
Folk dances States 4. Notable mudras include
Wanawan Rajasthan 'Katakamukha Hasta', symbolizing
Panihari Jammu and Kashmir 'Om', and 'Ekcharya lasyam' style,
Lavani Gujarat where one dancer portrays multiple
Dandiya Maharashtra roles.

Select the correct code: Using the information provided in the


statements above, please select the option
a) 1-B, 2-A, 3-D, 4-C
that accurately corresponds to the given
b) 1-A, 2-B, 3-D, 4-C
information:
c) 1-B, 2-A, 3-C, 4-D
d) 1-A, 2-B, 3-C, 4-D a) Kuchipudi
b) Kathakali
c) Mohiniyattam
Q.47) Which of the following Dance forms d) Bharatanatyam
are considered as Classical Dance forms in
India?
Q.49) With reference to the Kathakali,
1. Bharatanatyam
consider the following statements:
2. Kuchipudi
3. Kathakali 1. It is a theatrical art form that
4. Bihu integrates music, dance, and
5. Sattriya drama, emerged from the roots of
6. Garbha folk drama tradition.
7. Manipuri 2. It is primarily enacted by troupes
consisting of both male and female
Select correct code:
performers.
a) 1, 2, 3 and 5 only 3. It is performed in open-air theaters
b) 1, 2, 3, 4 and 7 only or temple premises with the natural
c) 1, 2, 3, 5, and 7 only
.
IASBABA’S PRELIMS TEST SERIES (AIPTS) 2024 – TEST 7 (ART & CULTURE)

surroundings and brass lamps 4. Garba


providing the backdrop. 5. Dandiya Raas

Which of the given above statements is/are Select the correct answer by using the code
correct? below:
a) 1 and 3 only a) 1, 2, 4 and 5 only
b) 1 and 2 only b) 2, 3, 4 and 5 only
c) 2 and 3 only c) 1, 3, 4 and 5 only
d) 1, 2 and 3 d) 1, 2, 3 and 4 only

Q.50) Recently ‘Bhutaradhana’ is in news Q.52) “It is Theatre traditions which is


with respect to this consider the following practiced in Karnataka and parts of Kerala.
statements: It is believed to be one of the oldest theatre
1. Bhutaradhana is a traditional traditions. Its origins can be traced back to
practice that involves worshiping the royal courts of the Vijayanagara Empire
divine beings and ancestral spirits, and it was performed by a specific
as well as different avatars of Hindu community called Jakkula Varu. Initially, it
Gods. involved a single artist performing a
2. This ritual is predominantly descriptive dance-drama, but later evolved
practiced in the coastal regions of into a more elaborate dance drama form.
Karnataka. The influence of the Vaishnava Bhakti
3. During Bhutaradhana, the Movement is strongly evident in the
performers adorn elaborate traditions”.
headgear and vibrant costumes as Which theater tradition from the following
part of their portrayal. options is described in the passage above?
Which of the given above statements is/are a) Chavittu Natakam
correct? b) Kuruvanji
a) Only One c) Yakshagana
b) Only Two d) Pragati Veshaalu
c) None
d) All Three
Q.53) Among the following Sanskrit plays
and authors, which pairing does not
Q.51) Which of the subsequent folk dances accurately matched?
are originated in the state of Gujarat?
a) Swapnavasavadatta - Bhavabhuti
1. Rathwa ni Gher b) Nagananda - Harshavardhana
2. Tippani c) Vikramorvasiyam - Kalidas
3. Charba d) Mricchakatika – Sudraka
.
IASBABA’S PRELIMS TEST SERIES (AIPTS) 2024 – TEST 7 (ART & CULTURE)

Q.54) With reference to the Indian d) None


Puppetry, consider the following
statements:
Q.56) With reference to ‘Kalaripayattu’,
1. Ravanchhaya, which comes from
one of the ancient martial arts in India,
Odisha, is a widely recognized type
consider the following statements:
of shadow puppetry that people
love to watch for entertainment. 1. Kalaripayattu originated in Kerala
2. Ravanchhaya utilizes deer skin around the 3rd century BC and is
puppets. now practiced throughout
3. Ravanchhaya Puppets often Southern India.
represent non-human characters 2. Kalaripayattu does involve
like trees and animals in the art drumming and singing, as they are
form. integral components alongside the
fighting style.
Which of the given above statements is/are 3. In the Haryana's Panchkula edition
correct?
of the Khelo India Youth Games, the
a) Only One Sports Ministry announced the
b) Only Two inclusion of Kalaripayattu.
c) None
Which of the given above statements is/are
d) All Three
correct?

a) 1 and 3 only
Q.55) With reference to the Indian Circus, b) 2 and 3 only
consider the following statements: c) 1 and 2 only
d) 1, 2 and 3 only
1. The first modern Indian circus was
founded by Prof. Vishnupant
Chatre, an accomplished
Q.57) With reference to the Martial arts in
equestrian and singer.
India, consider the following statements:
2. Chirakkara, emerged as the
epicentre of the circus revolution in 1. Thang-ta and Sarit Sarak are martial
India. arts forms that originated from the
3. West Bengal earned the reputation Meitei people of Manipur.
of being the 'cradle of the Indian 2. Thang-ta is an armed martial art,
circus'. while Sarit Sarak is an unarmed art
form emphasizing hand-to-hand
Which of the given above statements is/are combat.
correct?
3. Under British rule, these martial art
a) All Three forms were banned after the British
b) Only One took control of the region.
c) Only Two
.
IASBABA’S PRELIMS TEST SERIES (AIPTS) 2024 – TEST 7 (ART & CULTURE)

Which of the given above statements is/are 3. Mumbai


correct? 4. Bangalore
5. Srinagar
a) Only One
6. Chennai
b) Only Two
c) None Select the correct answer by using the code
d) All Three below:

a) 1, 2, 3, 4 and 5 only
b) 1, 2, 3, 5 and 6 only
Q.58) With reference to the ‘Cultural
c) 1, 2, 3, and 5 only
History of India’, consider the following
d) 1, 2, 3, 4, 5 and 6
statements:

4. Chhau dance, an eastern Indian


style, portrays scenes from epics Q.60) With reference to the Mohiniyattam,
like the Mahabharata and consider the following statements:
Ramayana, along with local folklore
1. Mohiniyattam incorporates
and abstract concepts.
elements of Nritta and Nritya,
5. Female dancers blend dance and
portraying Vishnu's feminine
martial arts, incorporating mock
dance.
combat techniques, as they
2. The graceful movements of
perform the Chhau dance at night
Mohiniyattam are accompanied by
in open spaces.
Hindustani music, with lyrics in
6. Chhau dance was added to
Manipravalam.
UNESCO's Representative List of
3. In Mohiniyattam, musical
Intangible Cultural Heritage of
accompaniment includes
Humanity in 2010.
instruments like flute, veena, and
Which of the given above statements is/are percussion instruments such as
correct? Mridangam, Maddalam, Idakka,
and Kuzhitalam.
a) 1 and 3 only
b) 2 and 3 only Which of the given above statements is/are
c) 1 and 2 only correct?
d) 1, 2 and 3 only
a) Only Two
b) Only One
c) All Three
Q.59) Which of the following cities is/are
d) None
included in the UNESCO Creative Cities
Network (UCCN)?

1. Varanasi
2. Hyderabad
.
IASBABA’S PRELIMS TEST SERIES (AIPTS) 2024 – TEST 7 (ART & CULTURE)

Q.61) With reference to the fusion of Which of the given above statements is/are
classical and folk music ‘Qawwali’, consider correct?
the following statements:
a) 1 and 3 only
1. Qawwali, a devotional music form, b) 2 and 3 only
is known for incorporating multiple c) 1 and 2 only
ragas and is predominantly written d) 1, 2 and 3
in Urdu.
2. Qawwali performances take place
in Sufi shrines, with solo or group Q.63) Consider the following statements
singers accompanied by a team. with respect to the Ramappa temple a
UNESCO World Heritage Site:
Which of the given above statements is/are
correct? 1. The architectural design of the
temple embraces the stellate form
a) 1 only
and incorporates purely Dravida
b) 2 only
style of vimana, which was made
c) Both 1 and 2
popular by the Pallavas.
d) Neither 1 nor 2
2. Ramappa temple is a sandstone
temple dedicated to Shiva.
3. The Kakatiyas showcased their skill
Q.62) With reference to the differences
in construction and geotechnology
between Hindustani music and Carnatic
by utilizing sand in the foundations
music, consider the following statements:
to construct buildings that could
1. Hindustani music reflects Persian withstand earthquakes.
and Islamic influences, while
Which of the given above statements is/are
Carnatic music is rooted in ancient
correct?
Indian musical systems.
2. Hindustani music follows a a) Only One
structured approach with defined b) Only Two
compositions and strict rules for c) All Three
improvisation, while Carnatic music d) None
emphasizes improvisation,
providing flexibility and exploration
in ragas. Q.64) With reference to the Warli
3. Hindustani music utilizes sitar, paintings, consider the following
sarod, tabla, harmonium, and flute, statements:
while Carnatic music incorporates
1. Warli paintings were primarily
veena, violin, mridangam, ghatam,
found along the Gujarat-
and kanjira as accompanying
Maharashtra border.
instruments.
.
IASBABA’S PRELIMS TEST SERIES (AIPTS) 2024 – TEST 7 (ART & CULTURE)

2. Warli paintings do not portray How many pairs is/are correctly matched?
mythical tales but instead centre on
a) One pair only
everyday life and its various
b) Two pairs only
aspects.
c) Three pairs
3. Warli wall paintings utilize basic
d) None of the above
geometric shapes such as triangles,
circles, squares, dots, and dashes.

Which of the given above statements is/are Q.67) With reference to the Pala and Sena
correct? School of Architecture, consider the
following statements:
a) 1 and 3 only
b) 2 and 3 only 1. The Palas, Buddhist rulers, showed
c) 1 and 2 only tolerance and constructed Viharas,
d) 1, 2 and 3 Chaityas, and Stupas, while Senas
built the Hindu temples and
promoted Buddhist architecture,
Q.65) Consider the following Ashokan resulting in a fusion of religious
inscriptions: influences in the region's
architecture.
1. Rummindei Pillar Inscription
2. The architectural style with curved
2. Bhabhru Edict
or sloping roofs resembling
3. Schism Edicts
bamboo huts, known as the
4. Nigalisagar Pillar Inscription
"Bangla roof," was later adopted by
Among the inscriptions mentioned above, Mughal architects.
which one signifies Ashoka's personal 3. The Sena rulers were associated
interest in Buddhism? with prominent universities such as
Nalanda, Jagaddala, Odantapuri,
a) 1, 2 and 3 only and Vikramshila.
b) 2, 3 and 4 only 4. Monument Dhakeshwari Temple is
c) 1, 2 and 4 only associated with the Pala rulers.
d) 1, 2, 3 and 4
Which of the given above statements is/are
incorrect?
Q.66) With reference to the Sun temples a) Only one
in India, consider the following pairs: b) Only Two
1. Brahmanya Dev Madhya c) Only Three
Temple Pradesh d) All Four

2. Dakshinaarka Temple Bihar

3. Martand Sun Temple Tamil Nadu


.
IASBABA’S PRELIMS TEST SERIES (AIPTS) 2024 – TEST 7 (ART & CULTURE)

Q.68) Consider the following statements c) 1 and 2 only


with respect Dashavatara Temple, d) 1, 2 and 3 only
Deogarh:

1. The temple is thought to have been


Q.70) With reference to the ‘Shadow
built around the early sixth century
puppetry’, consider the following
CE.
statements:
2. The temple adopts the
Panchayatana style with a central 1. Flat figures made of cut-out leather
shrine and four smaller corner are used for shadow puppetry.
shrines 2. Single side of the leather figures are
3. The temple is devoted to the painted identically.
worship of Lord Vishnu 3. Togalu gombeyatta is a notable
example of shadow puppetry.
Which of the given above statements is/are
correct? Which of the given above statements is/are
correct?
a) 1 and 3 only
b) 2 and 3 only a) Only Two
c) 1 and 2 only b) Only One
d) 1, 2 and 3 only c) All Three
d) None of the above

Q.69) With reference to the official


language for the states, consider the Q.71) Languages in India can be classified
following statements: into major subgroups with respect to this
consider the following statements in this
1. The Constitution does not prescribe regard:
a specific official language for the
States to use in their official 1. The majority of languages spoken in
activities. India belong to the Indo-Aryan
2. The States have the autonomy to group.
select their preferred official 2. Assamese belong to the Sino-
language. Tibetan group of languages.
3. The official language chosen by the 3. Austric group of languages are
States must be one among the older than Indo-Aryan languages.
languages listed in the Eighth Which of the above statements is/are
Schedule of the Constitution. correct?
Which of the above given statements is/are a) Only One
correct? b) Only Two
a) 1 and 3 only c) All Three
b) 2 and 3 only d) None of the above
.
IASBABA’S PRELIMS TEST SERIES (AIPTS) 2024 – TEST 7 (ART & CULTURE)

Q.72) Consider the following statements Q.74) Consider the following statements
with reference to sects under Vaishnavism: with reference to Ajivikas sect:

1. The varkari sect, known for its 1. They were considered as one of the
devotion to Lord Vitthala, primarily Shramana schools.
worships in the temple town of 2. They embraced the doctrine of
Pandharpur in Maharashtra. karma.
2. The Ramanandi Sampradaya traces 3. Savatthi (Shravasti) in Uttar Pradesh
its lineage back to the prominent is believed to be the centre of
saint and poet, Ramanujacharya. Ajivika.
3. The Brahma Sampradaya
Which of the above statements is/are
emphasizes the philosophy of
correct?
Dvaitha (dualism).
a) Only One
Which of the above statements is/are
b) Only Two
correct?
c) None of the above
a) All Three d) All Three
b) Only One
c) Only Two
d) None of the above Q.75) With reference to the Bodhisattva in
Buddhism, consider the following
statements:
Q.73) With reference to Zoroastrianism
1. The concept of bodhisattva is
consider the following statements:
central to Hinayana sect of
1. It is a monotheistic religion which Buddhism.
believes in one eternal god. 2. Avalokitesvara, Vajrapani and
2. They emphasize the existence of a Manjusri are the three protective
supreme deity named Angra deities around Buddha.
Mainyu. 3. Manjushri Bodhisattva is associated
3. Their sacred text is called Zend with wisdom and insight.
Avesta.
Which of the following statements is/are
Which of the above statements is/are correct?
correct?
a) 1 and 2 only
a) 1 and 2 only b) 1 and 3 only
b) 1 and 3 only c) 2 and 3 only
c) 2 and 3 only d) 1, 2 and 3
d) 1, 2 and 3
.
IASBABA’S PRELIMS TEST SERIES (AIPTS) 2024 – TEST 7 (ART & CULTURE)

Q.76) Which of the following is/are not the 2. It begins on 22nd March every year.
salient features of the Vajrayana 3. It was adopted as the National
Buddhism? Calendar in the year 1957

1. It involved merging Brahmanical Which of the above statements is/are


rites with Buddhist philosophies. correct?
2. It is based on Mahayana Buddhist
a) Only One
philosophy.
b) Only Two
3. It’s predominant in Nepal, Bhutan
c) All Three
and Mongolia.
d) None of the above
Select the correct answer using the code
given below:
Q.79) Consider the following statements:
a) 1 and 2 only
b) 1 and 3 only 1. Sangeet Natak Academy is the first
c) 2 and 3 only national academy for preserving
d) 1, 2 and 3 and promoting performing arts in
India.
2. It is purely the central agency that
Q.77) Consider the following statements monitors the preservation of our
regarding Buddhist literature: cultural heritage.

1. Suttapitaka is a compilation of Which of the above statements is/are


sayings of Buddha. correct?
2. Vinaypitaka is a compilation of rules
a) 1 only
of Sangha.
b) 2 only
3. Abhidhamma pitaka is a
c) Both 1 and 2
compilation of philosophies of
d) Neither 1 nor 2
Dhamma.

Which of the above statements are


correct? Q.80) “It is a colourful and vibrant festival
celebrated by women with flowers that
a) Only One
grow exclusively in each region. Different,
b) Only Two
unique seasonal flowers are arranged in
c) None of the above
concentric layers in the shape of a temple
d) All Three
gopuram thus forming a beautiful flower
stack. The festival celebrates the inherent
relationship between earth, water, and
Q.78) With reference to the Saka Calendar,
human beings”.
consider the following statements:

1. It has the same number of months Which among the following festival is
described in the above passage?
like Vikram era.
.
IASBABA’S PRELIMS TEST SERIES (AIPTS) 2024 – TEST 7 (ART & CULTURE)

a) Mahapushkaram a) 1 only
b) Bathukamma b) 2 only
c) Makaravilakku c) Both 1 and 2
d) Attukal d) Neither 1 nor 2

Q.81) With reference to Vedic literature, Q.83) Consider the following statements:
consider the following statements:
1. Upnayana ceremony was
1. The Atharva Veda deals with the conducted before the learning
peace and prosperity of the human process began for any student.
society, and is concerned with the 2. Charaka Samhita is an important
daily life of man. work on surgery.
2. The Puranas were written to 3. Ghatika refers to an irrigational
illustrate and expound the truth of device used during the 7th and the
the Vedas. 8th centuries.
3. The Upanishads is the literature in
Which of the statements given above is/are
which ancient sages realised that in
correct?
the final analysis, man has to know
himself. a) Only One
b) Only Two
Select the correct answer using the code
c) All Three
given below:
d) None of the above
a) Only One
b) Only Two
c) All Three Q.84) Which of the following statements
d) None of the above is/are correct about Puranas?

1. The Puranas narrate the incidents


of various influential kings.
Q.82) Consider the following statements
2. It focuses on the divine Hindu
with reference to education during
trinity Gods: Brahma, Vishnu and
medieval India:
Mahesh.
1. Makhtabs were the centres of 3. The Puranas discuss about the
elementary education whereas geography of the world thus helps
Madrasas were educational in understanding of the physical
institutions for higher education features.
during the medieval period.
Select the correct answer using the code
2. Muizzi was a famous madarasa in
given below:
Bidar, Karnataka.
a) 1 and 2 only
Which of the following statement(s) is/are
b) 2 only
correct?
.
IASBABA’S PRELIMS TEST SERIES (AIPTS) 2024 – TEST 7 (ART & CULTURE)

c) 2 and 3 only Q.87) Consider the following statements


d) 1, 2 and 3 regarding Ancient Indian Literature:

1. Lord Buddha used Pali language in


his sermons.
Q.85) Arrange the following literary works
2. Jain literature was primarily written
in chronological order starting from the
in Prakrit having moral instructions.
earliest:
3. Panchatantra was mainly written in
1. Rig Veda Hindi Language.
2. Natyashashtra
Which of the above statements is/are
3. Vedangas
correct?
4. Atharvaveda
a) 1 and 2 only
Select the correct answer using the code
b) 1 and 3 only
given below:
c) 2 and 3 only
a) 1-2-3-4 d) 1, 2 and 3
b) 1-4-3-2
c) 1-2-4-3
d) 1-4-2-3 Q.88) Consider the following statements
about a Charvaka Philosophy:

1. It has rejected the idea of the


Q.86) Which of the following pairs is/are
existence of God.
incorrectly matched?
2. It recognizes four material
Bhakti Movement Literary Language elements as the substance: earth,
1. Namdev Marathi water, fire, and air.
2. Tulsidas Avadhi 3. It rejected the notion of an after
3. Kabir Bhojpuri world, karma, liberation (moksha).
4. Narsi Mehta Bangla
Which of the following School of
Philosophy best describes the above
Select the correct answer using the code
characteristic features?
given below:
a) 1 and 2 only
a) 2 only
b) 1 and 3 only
b) 1 and 4 only
c) 2 and 3 only
c) 2 and 4 only
d) 1, 2 and 3
d) 4 only
.
IASBABA’S PRELIMS TEST SERIES (AIPTS) 2024 – TEST 7 (ART & CULTURE)

Q.89) “This school believes in the Q.91) With reference to Central Board of
physicality of the Universe and is Film Certification, consider the following
considered to be the realistic and objective statements:
philosophy that governs the universe. This
1. It is a statutory body under the
school has a scientific approach and
Ministry of Information and
explains explain the phenomenon of this
Broadcasting.
universe. It was also responsible for the
2. It is not necessary to obtain CBFC
beginning of physics in the Indian
certification for imported foreign
subcontinent”.
films in India.
Which of the following Schools of 3. There is exception to the CBFC
Philosophy is explained in the above certificate for films made especially
passage? for Doordarshan.

a) Samkhya How many of the above given statements


b) Vedanta is/are correct?
c) Mimamsa
a) One only
d) Vaisheshika
b) Two only
c) Three
d) None of the above
Q.90) With reference to various cultural
institutions in India, consider the following
statements:
Q.92) In which of the following state Teesta
1. The Archaeological Survey of India Rangit Tourism Festival is celebrated:
is the premier institution for the
a) Sikkim
preservation of tangible as well as
b) West Bengal
intangible cultural heritage in India.
c) Odisha
2. The Indira Gandhi National Centre
d) Assam
for Arts is an autonomous
organization concentrating on
visual and performing arts.
Q.93) With reference to the ‘Hornbill
3. The Centre for Cultural Resources
Festival’, consider the following
and Training is responsible for
statements:
linking education with culture.
1. The festival coincides with the
Which of the above statements is/are
statehood day of Nagaland.
correct?
2. It is celebrated in the first week of
a) Only One November
b) Only Two 3. The Great Hornbill is the state bird
c) All Three of Nagaland.
d) None of the above
.
IASBABA’S PRELIMS TEST SERIES (AIPTS) 2024 – TEST 7 (ART & CULTURE)

How many of the above given statements a) Only One


is/are correct? b) Only Two
c) All Three
a) One only
d) None
b) Two only
c) Three
d) None of the above
Q.96) Where does the ‘Sair-e-Gul Faroshan’
festival held?
Q.94) With reference to science and a) Shimla
technology in ancient India, consider the b) Mussoorie
following statements: c) Delhi
1. Aryabhatta calculated the value of d) Hyderabad
‘Pi’.
2. Kanad gave the atomic theory.
3. Charak was Raj Vaidya (royal Q.97) Kheer Bhawani is a?
doctor) in the court of Kanishka. a) Geographical tag holding local
4. Bhaskaracharya introduced sweet delicacy of Madhya Pradesh
negative numbers and operations b) Prestigious Saree variant of Kerala
on zero into mathematics. weaved from banana leaves.
Which of the following statements is/are c) Annual festival of Jammu and
correct? Kashmir dedicated to the Goddess
Ragnya Devi.
a) Only One d) Famous painting of Mughal style
b) Only Two
c) Only Three
d) All Four Q.98) Consider the following statements
regarding Fairs and Festivals in India:

Q.95) Consider the following statements 1. Pongal is the harvest festival of


on Kumbh Mela: Kerala.
2. Makar Sankranti is a harvest festival
1. It is the largest peaceful which marks the beginning of Sun’s
congregation of pilgrims in the journey from Southern Hemisphere
world. to Northern Hemisphere.
2. The Ardh-Kumbh Mela takes place 3. Gangasagar Mela is the most
every six years in Haridwar and important fair celebrated in West
Prayagraj. Bengal.
3. The Chinese traveller Hiuen Tsang
described this ritual. Which of the above statements is/are
correct?
Which of the above statements is/are
correct? a) Only One
.
IASBABA’S PRELIMS TEST SERIES (AIPTS) 2024 – TEST 7 (ART & CULTURE)

b) Only Two authored and published by an


c) All Three Indian citizen.
d) None
Select the correct answer using the code
given below:

Q.99) Consider the following statements a) 1 and 2 only


related to the coinage of India: b) 1 only
c) 2 only
1. The Gupta Empire produced large
d) 2 and 3 only
numbers of gold coins depicting the
Gupta kings performing rituals.
2. Kushan coins contained inscriptions
of Siva, Buddha and Kartikeya.
3. The Sultans of Delhi introduced
Islamic Calligraphy on coins for the
first time in India.

Which of the above statements is/are


correct?

a) Only One
b) Only Two
c) None
d) All Three

Q.100) Which of the following statements


regarding different cultural awards is/are
correct?

1. Jnanpith Award is bestowed only to


Indian writers writing in one of the
22 languages mentioned in the
Eighth Schedule of Indian
Constitution.
2. Tagore Award is given for
outstanding achievement in
fostering harmony and
universalism and values of cultural
harmony.
3. Saraswati Award is given to
outstanding literary work in Hindi,
.
IASBABA’S PRELIMS TEST SERIES (AIPTS) 2024 – TEST 7 (ART & CULTURE) SOLUTIONS

Q.1) Solution (a)

Explanation:

Ancient Scripts of India

• A script is also known as a writing system or orthography. It is a standard for


representing the parts of a spoken language by making specific marks on a medium
(paper, rocks, birch- bark, etc.). The two ancient scripts in India comprise the Brahmi
script and the Kharosthi script. Most of the ancient and modern scripts in India are
developed from the Brahmi script be it Devanagari, Tamil, Telugu, Kannada, Odia,
Assamese/Bengali, etc. Hence, it can very muchbe maintained that Brahmi is the
mother of scripts.
• However, Urdu is written in a script derived from Arabic, and a few minor languages,
such as Santhali, use independent scripts.

Indus Script

• The Indus Script is a corpus of symbols produced by the Indus Valley Civilisation. Most
inscriptions are extremely short and it is unclear if these symbols constitute a script
used to record a language. (Hence statement 1 is correct)

Brahmi Script

• Brahmi is the oldest writing system used in the Indian sub- continent and in Central
Asia during the final centuries BC and the early centuries AD. Some believe that Brahmi
was derived from a contemporary Semitic Script or perhaps the Indus Script. All
surviving Indic Scripts in South East Asia are descendants of Brahmi.
• The best-known Brahmi inscriptions are the rock-cut edicts of Ashoka in North-Central
India, dated 250-232 BC. The script was deciphered in 1837 by James Prinsep. (Hence
statement 2 is correct)
• Brahmi is usually written from left to right and is an abugida, meaning that each unit
is absed on a consonant and the vowel notation is secondary, except when the vowels
commence a word.

Kharosthi Script

• The Kharosthi Script (3rd century BC- 3rd century AD) is an ancient script used in
ancient Gandhara (present day Afghanistan and Pakistan) to write the Gandhari Prakrit
and Sanskrit. It is a sister script of Brahmi and was also deciphered again by James
Prinsep.
• Kharosthi is also anabugida like Brahmi. Kharosthi includes a set of numerals that are
similar to Roman numerals like I, X, etc. The Kharosthi script is mostly written right to
left, but some inscriptions also show the left to right direction of Kharosthi. (Hence
statement 3 is incorrect).
.
IASBABA’S PRELIMS TEST SERIES (AIPTS) 2024 – TEST 7 (ART & CULTURE) SOLUTIONS

Kaithi Script

• It is a historical Brahmic script that was used widely in parts of Northern and Eastern
India, primarily in the present-day states of Uttar Pradesh, Jharkhand and Bihar. In
particular, it was used for writing legal, administrative and private records. The script
was widely used during the Mughal period.

Q.2) Solution (b)

Explanation:

• The Harappan civilization utilized a wide range of materials for crafting ornaments,
including valuable metals, gemstones, bones, and baked clay.
• Both men and women adorned themselves with various types of ornaments, such as
necklaces, headbands, armlets, and finger rings. However, certain items like AP
Girdles, earrings, and anklets were exclusively worn by women. (Hence statement 3 is
incorrect)
• Beads crafted from carnelian, amethyst, quartz, and steatite were highly popular and
mass-produced, as evidenced by the discovered factories in Chanhudaro and Lothal.
• When it came to textiles, the Harappans employed both cotton and wool, which were
spun by individuals from all social classes. (Hence statement 1 is correct)
• Furthermore, the people of that era demonstrated a keen sense of fashion, as
indicated by their diverse hairstyles and beard styles. (Hence statement 2 is correct)

Q.3) Solution (b)

Explanation:

Basis Ashokan pillars Iranian pillars


Composition The Ashokan pillars were composed of The shafts of Iranian pillars
monolithic shafts, meaning they were were constructed using
carved from a single piece of stone, multiple pieces of sandstone
primarily chunar sandstones joined together with cement
(Hence statement 1 is
incorrect)
Location The Ashokan pillars were erected The Iranian pillars were
independently affixed to state buildings.
(Hence statement 2 is
correct)
.
IASBABA’S PRELIMS TEST SERIES (AIPTS) 2024 – TEST 7 (ART & CULTURE) SOLUTIONS

Q.4) Solution (b)

Explanation:

Sculptures played a central role in adorning stupas, as well as in the construction of toranas
and medhis, serving as a powerful medium for religious expression. Among the renowned
sculptures from the Mauryan period, the figures of Yaksha and Yakshi stand out. These
sculptures held great religious significance and were objects of veneration within Jainism,
Hinduism, and Buddhism. The earliest reference to yakshi can be traced back to
Silappadikaram, a Tamil text, while each of the Jain Tirthankars was connected to a yakshi
figure.

Shalabhanjika

• Within Buddhist art, the Shalabhanjika represents a yakshi figure gracefully holding a
tree or branch. These Shalabhanjika depictions can be observed as decorative figures
on the gateways of the renowned Great Stupa in Sanchi. (Hence statement 1 is
incorrect)
• The motif of the Shalabhanjika was considered a symbol of good fortune and was
incorporated into the embellishment of the stupa. (Hence statement 2 is correct)
• This suggests that numerous individuals who embraced Buddhism also infused it with
their pre-Buddhist or non-Buddhist traditions and beliefs, contributing to its richness
and diversity.

Q.5) Solution (b)

Explanation:

Gandhara School

• The Gandhara school of art emerged in the western frontiers of Punjab, near present-
day Peshawar and Afghanistan.
• Greek invaders introduced the artistic traditions of Greek and Roman sculptors, which
influenced the local art of the region. (Hence statement 1 is correct)
• As a result, the Gandhara School became known as the Greco-Indian school of art.
• It was patronized by Kushana rulers
• The Gandhara School thrived in two phases between 50 BC and 500 AD.
• The earlier phase of the school utilized bluish-grey sandstone for sculpting.
• In the later phase, mud and stucco became the preferred materials for sculpting.
• The images of Buddha and Bodhisattvas in the Gandhara school were inspired by the
Greco-Roman pantheon and bore a resemblance to Apollo.
• The depiction of Buddha showed him in a spiritual state with wavy hair, adorned with
fewer ornaments, and seated in a yogic posture. (Hence statement 2 is correct)
.
IASBABA’S PRELIMS TEST SERIES (AIPTS) 2024 – TEST 7 (ART & CULTURE) SOLUTIONS

• The eyes of Buddha sculptures were typically half-closed, reflecting a meditative state.
• A protuberance on the head of Buddha sculptures symbolized his omniscience.

In the Mathura School, the Buddha is depicted seated in the lotus position, exhibiting
different mudras or hand gestures. His countenance exudes an aura of grace and serenity.
(Hence statement 3 is incorrect)

Q.6) Solution (b)

Explanation:

Mathura School Amaravati School

It was indigenous in development and free It was an indigenous development and


from external cultural influences. (Hence remained unaffected by external cultures.
statement 1 is incorrect)
The sculptures of the Mathura school were The sculptures of the Amaravati school were
crafted using distinctive spotted red crafted from white marble. (Hence
sandstone. statement 2 is correct)
The art of the time was influenced by The predominant influence on this art style
Hinduism, Jainism, and Buddhism, was Buddhist. (Hence statement 3 is
representing all three religions. correct)
The Kushana rulers provided patronage for The Satvahana rulers provided patronage for
this art form. the development of this art form.
The art flourished in and around Mathura, The art flourished in the Krishna-Godavari
Sonkh, and Kankali Tila in Uttar Pradesh, lower valley, specifically in and around
with Kankali Tila being particularly Amaravati and Nagarjunakonda in Andhra
renowned for its Jain sculptures. Pradesh.
The Buddha is depicted in a joyous state with The sculptures primarily focus on narrative
a smiling countenance. The sculpture art, placing less emphasis on the individual
showcases a muscular body adorned in features of Buddha. They typically depict the
fitted attire. The face and head are cleanly life stories of Buddha and Jataka tales,
shaven. The Buddha is seated in the lotus showcasing the previous lives of Buddha in
position, displaying various mudras, while both human and animal forms.
his face emanates grace. A similar cranial
protrusion is also portrayed.
.
IASBABA’S PRELIMS TEST SERIES (AIPTS) 2024 – TEST 7 (ART & CULTURE) SOLUTIONS

Q.7) Solution(c)

Explanation:

Various Mudras related to Buddha

Bhumisparsha Mudra

• One of the most common mudras found in statues of Buddha.


• It depicts the Buddha sitting in meditation with his left hand, palm upright, in his lap,
and his right hand touching the earth.
• This mudra is commonly associated with blue Buddha known as Akshobhya.
• Significance: 'Calling the Earth to Witness the Truth, and it represents the moment of
Buddha attaining enlightenment.

Dhyana Mudra

• It indicates meditation and is also called 'Samadhi' or 'Yoga' mudra.


• It depicts Buddha with both hands in the lap, back of the right hand resting on the
palm of the left hand with fingers extended. In many statues, the thumbs of both
hands are shown touching at the tips, thus forming a mystic triangle.
• It signifies attainment of spiritual perfection.
• This mudra was used by Buddha during the final meditation under the Bodhi tree.

Vitarka Mudra

• It indicates teaching and discussion or intellectual debate.


• The tips of the thumb and index finger touch each other, forming a circle. The right
hand is positioned at shoulder level and the left hand at the hip level, in the lap, with
palm facing upwards.
.
IASBABA’S PRELIMS TEST SERIES (AIPTS) 2024 – TEST 7 (ART & CULTURE) SOLUTIONS

• It signifies the teaching phase of preaching in Buddhism. The circle formed by the
thumb and index finger maintains the constant flow of energy, as there is no beginning
or end, only perfection.

Abhaya Mudra

• It indicates fearlessness and symbolises strength and inner security.


• The right hand is raised to shoulder height with arm bent. The palm of the right hand
faces outwards and the fingers are upright and joined. The left hand hangs downwards
by the side of the body.
• This gesture was shown by Buddha Immediately after attaining enlightenment.

Dharmachakra Mudra

• It means Turning the Wheel of the Dharma or Law, Le. Setting into motion the wheel
of Dharma
• This mudra involves both hands. The right hand is held at chest level with the palm
facing outwards. A mystic circle is formed by joining the tips of the index finger and
the thumb. The left hand is turned inward and the index finger and thumb of this hand
join to touch the right hand's circle.(Hence option c) is correct)
• This gesture was exhibited by Lord Buddha while he preached the first sermon to a
companion after his enlightenment in the Deer Park of Sarnath.

Anjali Mudra

• This mudra signifies greetings, devotion and adoration.


• Both hands close to the chest, palms and fingers join against each other vertically.
• It is common gesture used in India to greet people (Namaste), It signifies adoration of
the superior and is considered a sign of regards with deep respect.

Uttarabodhi Mudra

• It means supreme enlightenment.


• Holding both hands at the level of the chest, intertwining all the fingers except Index
fingers, extending index fingers straight up and touching each other.
• This mudra is known for charging one with energy. It symbolises perfection,
Shakyamuni Buddha (liberator of Nagas) presents this mudra.

Varada Mudra

• It indicates charity, compassion or granting wishes. The right arm is extended in a


natural position all the way down, with the palm of the open hand facing outwards
towards onlookers. If standing, the arm is held slightly extended to the front. It can be
a left- hand gesture as well.
• Through the five extended fingers, this mudra signifies five perfections: generosity,
morality, patience, effort and meditative concentration.
.
IASBABA’S PRELIMS TEST SERIES (AIPTS) 2024 – TEST 7 (ART & CULTURE) SOLUTIONS

Karana Mudra

• It indicates warding off evil.


• The hand is stretched out, either horizontally or vertically, with the palm forward. The
thumb presses the folded two middle fingers but the index and little fingers are raised
straight upwards.
• It signifies expelling demons and negative energy. The energy created by this mudra
helps remove obstacles such as sickness or negative thoughts.

Vajra Mudra

• It indicates knowledge. This mudra is better known in Korea and Japan.


• In this mudra, the erect forefinger of the left hand is held in the fist of the right hand.
It is seen in the mirror-inverted form also.
• This mudra signifies the importance of knowledge or supreme wisdom. Knowledge is
represented by the forefinger and the fist of the right hand protects it.

Q.8) Solution (b)

Explanation:

Major Pillar edicts of Asoka

• Major Pillar edict 1: Ashoka's principle of protection of the people.(Hence pair 1 is


correctly matched)
• Major Pillar edict 2: Dhamma as a minimum of sins, virtues, compassion, truthfulness
and purity(Hence pair 2 is correctly matched)
• Major Pillar edict 3: Abolishes sins of harshness, cruelty, and anger. (Hence pair 3 is
incorrectly matched)
• Major Pillar edict 4: Duties of Rajukas
• Major Pillar edict 5: List of animals and birds which should not be killed on some
days,and another list of animals which should not be killed at all. Livinganimals must
not be fed with other living animals.
• Major Pillar edict 6: Dhamma policy of Ashoka
• Major Pillar edict 7: Works carried out by Asoka for Dhamma. In the words of Ashoka,
I have set up pillars of morality, appointed Mahamatras (officer of morality) and issued
proclamations on morality.

Q.9) Solution (b)

Explanation:
.
IASBABA’S PRELIMS TEST SERIES (AIPTS) 2024 – TEST 7 (ART & CULTURE) SOLUTIONS

• The rise of the Gupta Empire in the 4th century AD is often regarded as the 'Golden
Age of India,' characterized by significant cultural and artistic achievements. While
early Gupta rulers followed Buddhist architectural traditions, temple architecture
gained prominence during the later Gupta phase under the patronage of Hindu rulers.
• Temple construction reached its pinnacle during this era, showcasing remarkable
architectural feats. Additionally, the Gupta Age witnessed the zenith of Buddhist and
Jain art.(Hence statement 1 is correct and statement 2 is incorrect)
• In the later phase of Gupta rule, Brahmanical rulers came to power, yet they
demonstrated remarkable religious tolerance.
• Worship of three primary deities prevailed in different regions of India: Vishnu in the
northern and central parts, Shiva in the southern part, and Shakti in the eastern part
as well as along the Malabar Coast in the southwest of India.(Hence statement 3 is
correct)

Q.10) Solution (c)

Explanation:

Khajuraho School

In central India, the Chandela rulers developed a unique style of temple architecture, which
became known as the Khajuraho School or Chandela School. The temples of this school
exhibited the following characteristics:

• In these temples, both the interior and exterior walls were lavishly decorated with
carvings.
• The sculptures were generally erotic in their themes and drew inspiration from
Vatsyayana's Kama Sutra (Hence statement 1 is correct)
• The temples were constructed using sandstone as their primary material (Hence
statement 2 is correct)
• The temples consisted of three
chambers: Garbhagriha, Mandapa,
and Ardhamandapa.
• Some temples featured an entrance
vestibule known as Antrala, leading
to the Garbhagriha.
• The temples generally faced either
north or east
• The temples were elevated on relatively high platforms and belonged to both Hindu
and Jain religions (Hence statement 3 is correct)
• Notable examples include the Kandariya Mahadev Temple and the Lakshmana Temple
in Khajuraho.
.
IASBABA’S PRELIMS TEST SERIES (AIPTS) 2024 – TEST 7 (ART & CULTURE) SOLUTIONS

Q.11) Solution (d)

Explanation:

Ajanta Caves

• Ajanta is a collection of rock-cut caves located near Aurangabad in Maharashtra, India,


within the Sahyadri ranges and along the Waghora River. (Hence statement 1 is
correct)
• Among the 29 caves at Ajanta, 25 were utilized as residential Viharas, while 4 served
as prayer halls or Chaitya.
• These caves were developed between 200 BC and 650 AD, spanning a considerable
period.
• The cave paintings in Ajanta were executed using the fresco painting technique,
exhibiting a remarkable level of naturalism. Local vegetation and minerals were used
to obtain colors, with red outlines and subsequent internal painting. (Hence statement
2 is correct)
• Cave 16 stands out as a particularly elegant example of cave architecture within the
Ajanta site.
• The paintings within the caves primarily revolve around Buddhism, depicting scenes
from the life of Buddha and Jataka stories. (Hence statement 3 is incorrect)
• Out of the 29 caves, 5 were developed during the Hinayana phase, while the remaining
24 were created during the Mahayana phase of Buddhism.
• Buddhist monks inscribed the Ajanta caves under the patronage of the Vakataka kings,
with Harishena being a prominent ruler. (Hence statement 4 is correct)
• The Ajanta caves are referenced in the travel accounts of Chinese Buddhist travelers
Fa Hien and Hiuen Tsang.
• Notable sculptures within the Ajanta Caves include the Mahaparinirvana of Buddha in
Cave 26 and the depiction of a Naga king and his consort in Cave 19.

Q.12) Solution (b)

Explanation:

Nagara style of Temple Architecture

The Nagara style of architecture emerged in northern India from the 5th century AD onwards.
It is characterized by a distinct temple design. Within the Nagara school, different sub-schools
developed in the western, central, and eastern parts of the country.

Key features of the Nagara style include:


.
IASBABA’S PRELIMS TEST SERIES (AIPTS) 2024 – TEST 7 (ART & CULTURE) SOLUTIONS

• Temples typically followed the Panchayatana style, which involved subsidiary shrines
arranged in a cruciform layout around the main shrine. (Hence statement 1 is correct)
• Assembly halls or mandaps were often present in front of the main shrine.
• Outside the inner sanctum (garbhagriha), statues of the river goddesses Ganga and
Yamuna were placed.
• Generally, water tanks or reservoirs were not found within the temple premises.
(Hence statement 2 is incorrect)
• Temples were usually constructed on elevated platforms.
• The entrance to the temples featured pillared porticos.
• Shikharas were generally of three types:
o Latina or Rekha-prasad: They were square at the base, and the walls curve
inward to a point on the top.
o Phamsana: They had a broader base and were shorter in height than the Latina
ones. They slope upwards on a straight line.
o Valabhi: They had a rectangular base with the roof rising into vaulted
chambers. They were also called wagon-vaulted roofs.
• The top of the shikhara, the tower-like structure of the temple, had a vertical end that
terminated in a fluted disc called the Amalak. On top of the Amalak, a spherical shape
called the Kalash was placed.
• Inside the temple, the wall was divided into three vertical planes known as rathas,
forming triratha temples. Over time, temples with five, seven, and even nine vertical
planes, known as pancharatha, saptaratha, and navaratha temples, respectively,
emerged. These vertical planes served as panels for narrative sculptures.
• The ambulatory passageway or pradakshina path around the inner sanctum was
covered. (Hence statement 3 is correct)
• Generally, the temple premises did not feature elaborate boundary walls or gateways.

Q.13) Solution (a)

Explanation:

Architecture and Sculpture at Mahabalipuram

The ancient port city of Mamallapuram, named after Narasimhavarman I, who was also known
as Mamalla, thrived under the Pallava dynasty in Tamil Nadu. This 7th-century Pallava site was
recognized as a UNESCO World Heritage Site in 1984 under the designation 'Group of
Monuments at Mahabalipuram.' Some of the prominent architectural wonders included in
this group are as follows:
.
IASBABA’S PRELIMS TEST SERIES (AIPTS) 2024 – TEST 7 (ART & CULTURE) SOLUTIONS

• Ratha Temples or Pancha Ratha: Also known as Pandava Rathas, they are the earliest
rock-cut temples in India, comprising
Dharmaraja Ratha, Bhima Ratha, Arjuna
Ratha, Nakula and Sahadeva Ratha, and
Draupadi Ratha dated around the 7th
century AD. Dharmaraja Ratha is the largest
structure among the five.(Hence statement
1 is correct)
• Rock-cut Caves: They include the Varaha
Cave Temple, Krishna Cave Temple,
Panchapandava Cave Temple and the
Mahishasuramardini Mandapa (Bas-relief of Goddess Durga killing Mahishasura).
• Open-Air Rock Reliefs: They include Descent of the Ganges also known as Arjuna's
Penance or Bhagiratha's Penance carved on two huge boulders. It narrates the story
of descent of River Ganga on earth from heaven by the efforts of Bhagiratha. In close
proximity lies a big rock boulder, known as Krishna's Butter Ball.
• Shore Temple Complex: It has two small and one large temple enclosed within a two-
tier compound wall studded with images of Nandi, the Vahana of Shiva. The temple is
predominantly dedicated to Lord Shiva with a sculpture of Anantashayana Vishnu in
one of the three temples within the complex.(Hence statement 2 is correct)
• Ganesha Ratha: It is a stone temple dedicated to Lord Ganesha, in Mahabalipuram. It
was initially created to house a Shiva Linga, which was later removed. On it, there are
18 inscriptions in Grantha and Nagari scripts in the Sanskrit language.(Hence
statement 3 is incorrect)

Q.14) Solution (c)

Explanation:

The features of the Nataraja sculpture (Bronze Nataraja) of the Chola Periodare as follows

This elegant Nataraja sculpture is from the medieval period and originated in South India. It
was created under the patronage of the Chola Dynasty in the 12th century CE. Standing at a
height of 96.0 cm, with a width of 82.8 cm and a depth of 28.2 cm, it is currently showcased
in the Chola Bronzes gallery at the National Museum in New Delhi.

• The upper right hand holds the drum, which signifies the sound of creation. All
creations spring from the great sound of the damaruga.(Hence statement 1 is correct)
• The upper left hand holds the eternal fire, which represents destruction. Destruction
is the precursor and an inevitable counterpart of creation. (Hence statement 2 is
correct)
.
IASBABA’S PRELIMS TEST SERIES (AIPTS) 2024 – TEST 7 (ART & CULTURE) SOLUTIONS

• The lower right hand is raised in the gesture of abhaya signifying benediction and
reassuring the devotee not to be afraid.
• The lower left-hand points towards the upraised
foot and indicates the path of salvation.
• The left leg in bhujangatrasita stance represents
tirobhavn, that is kicking away the veil of maya or
illusion from the devotee's mind.
• Shiva is dancing on the figure of a small dwarf.
The dwarf or demon symbolizes ignorance and the
ego of an individual.(Hence statement 1 is correct)
• The matted and flowing locks of Shiva represent
the flow of River Ganges.
• In ornamentation, one ear of Shiva has a male
earring while the other has a female one. This represents the fusion of male and
female and is often referred to as Ardhanarishvara
• A snake is twisted around the arm of Shiva. The snake symbolizes the kundalini power,
which resides in the human spine in a dormant stage. If aroused, one can attain true
consciousness.
• The Nataraja is surrounded by a nimbus of glowing lights which symbolizes the vast
unending cycles of time. (Hence statement 3 is correct)

Q.15) Solution (a)

Explanation:

Prambanan Temple

• The UNESCO World Heritage Site in Indonesia houses the largest Hindu temple
complex, devoted to the Trimurti deities of the Hindu pantheon: Brahma (the Creator),
Vishnu (the Preserver), and Shiva (the Destroyer).(Hence statement 1 is incorrect and
statement 2 is correct)
• The temple compound encompasses approximately 240 smaller temples and shrines.
It was constructed in the 9th century AD by the Sanjaya kings of the Mataram or
Medang Kingdom, who primarily worshipped Shiva.(Hence statement 3 is incorrect)

Q.16) Solution (c)

Explanation:

• In Vijayanagara school they built more than one mandapa in each temple. The central
mandap came to be known as the kalyana mandapa (dedicated to divine marriage).
.
IASBABA’S PRELIMS TEST SERIES (AIPTS) 2024 – TEST 7 (ART & CULTURE) SOLUTIONS

The concept of secular buildings inside the temple premises was also introduced
during this period. (Hence statement 1 is correct)
• The major features of the architecture during Hoysala period was multiple shrines
were built around a central pillared hall. Unlike the crucified ground plan of the
Panchayatana style, the shrines led out in the shape of an intricately designed star. This
was known as the Stellate plan. (Hence statement 2 is correct)

Q.17) Solution (d)

Explanation:

Some of the features of Indo-Islamic architecture

• During the Indo-Islamic period, the use of arches and domes became prominent,
replacing the traditional Trabeate style of Saracenic architecture. This style was known
as the 'Arcuate' style.
• Minars, tall towers, were introduced around mosques and mausoleums by Islamic
rulers.
• Mortar was used as a cementing agent in their constructions.
• Indo-Islamic architecture avoided the depiction of human and animal figures.
• Compared to Hindu architecture, Indo-Islamic architecture introduced spaciousness,
massiveness, and breadth.
• Calligraphy and the Arabesque method were used for decoration. Arabesque involved
geometrical vegetal ornamentation, characterized by a continuous stem splitting into
leafy secondary stems in a decorative pattern.
• Geometry played a significant role in the architecture of this period, from decorative
patterns to achieving symmetry.
• Buildings featured intricate jaali works, which emphasized the importance of light in
Islamic religion.
• The Charbagh style of gardening, dividing a square block into four identical adjacent
gardens, was introduced by Islamic rulers. (Hence pair 1 is correctly matched)
• The pietra-dura technique was used for inlaying precious colored stones and gems into
walls for decoration. (Hence pair 2 is correctly matched)
• A unique feature was the use of the foreshortening technique, creating the illusion
that inscriptions appeared closer than they actually were.(Hence pair 3 is correctly
matched)

Q.18) Solution (b)

Explanation:
.
IASBABA’S PRELIMS TEST SERIES (AIPTS) 2024 – TEST 7 (ART & CULTURE) SOLUTIONS

• The Jaunpur School (1394-1479 AD) flourished under the patronage of the Sharqi
rulers, turning Jaunpur into a hub of art and cultural development. This architectural
style, known as the Sharqi style, shared similarities with the Pathan style and did not
incorporate minars. (Hence option b) is correct)
• One distinctive aspect of the buildings in this school is the utilization of large screens
adorned with striking and powerful characters in the central and side sections of the
prayer hall. An example of this style can be seen in the Atala Mosque located in
Jaunpur.

Q.19) Solution (c)

Explanation:

Malwa School

The Malwa School (1405-1569 AD) witnessed the cities of Dhar and Mandu in the Malwa
plateau emerging as significant centers of architecture. Notably, the buildings in this school
showcased a distinctive characteristic of employing diverse colored stones and marbles.

These structures featured spacious windows, possibly influenced by European styles, and
were embellished with stylized arches and pillars. Additionally, the stairs themselves were
utilized to enhance the overall aesthetics of the constructions. Interestingly, minars were not
included in this particular architectural style.

The Malwa School also referred to as the Pathan school of architecture, stands as a remarkable
example of environmental adaptation during that era due to the following elements:

• The incorporation of large windows ensured effective ventilation within the buildings
and rooms.
• The pavilions were gently arched, promoting airiness and helping maintain a cool
interior temperature during hot weather.
• Artificial reservoirs known as "baulis" were constructed within the premises for
water storage.
• Locally available materials were utilized in the construction process.
• The implementation of the batter system, introduced by the Tughlaqs, contributed to
the structural strength of the buildings.
• Some notable examples of this architectural style include the Rani Roopmati
Pavilion, Jahaz Mahal, and Ashrafi Mahal.

(Hence option c) is correct answer)

Q.20) Solution (b)


.
IASBABA’S PRELIMS TEST SERIES (AIPTS) 2024 – TEST 7 (ART & CULTURE) SOLUTIONS

Explanation:

Fatehpur Sikri

Fatehpur Sikri, created by Akbar, stands as a remarkable example of Indo-Islamic architecture.


Serving as a new capital city, it is often likened to a frozen moment in history due to its
distinctive blend of Hindu and Persian architectural styles. The buildings in Fatehpur Sikri
beautifully embody this fusion, creating a truly unique architectural marvel.

Some of the important buildings inside the city are as follows:

• BulandDarwaza the imposing 40-m red sandstone structure was built in 1575 AD, to
commemorate Akbar's victory over Gujarat. It is the highest gateway of the world.
• Salim Chishti's tomb was built in 1581 AD. It has beautiful Jaali work in white marble.
It has arabesque patterns with inscriptions of Quranic verses on the walls. It is in the
imperial complex which also includes BulandDarwaza and Jama Masjid. It was further
decorated by Jahangir in 1606 AD.
• Panch Mahal is a five-storeyed structure made of columns and is inspired by the
concept of a Persian badgir (wind-catcher). bed
• Jodha Bai's Palace or Mariam-uz-Zamani's Palace (Mary of the Age) has beautiful
interiors with Hindu motifs of bell and flowers.
• Ibadat Khana: Akbar used to meet the leaders of different religions here to have a
discussion.
• Hiran Minar was built in memory of Akbar's favourite elephant, named Hiran. It also
served as a lighthouse for travelers. It is uniquely designed, and its exterior wall
contains tusk like spikes.
• Red fort is located in Delhi. (Hence 6 is not located in Fathepur Sikri)

Q.21) Solution (b)

Explanation:

• Salim Chishti's tomb, built by Akbar in 1581 AD, showcases exquisite Jaali work in
white marble adorned with arabesque patterns and Quranic verses inscriptions on the
walls. It is located within the imperial complex, alongside Buland Darwaza and Jama
Masjid. Later, in 1606 AD, Jahangir enhanced its beauty with further embellishments.
(Hence statement 1 is incorrect)
• During Jahangir's reign, his wife Noor Jahan also took part in notable construction
projects. One remarkable example is the tomb of Itmad-ud-Daulah, honoring Noor
Jahan's father. This structure marked a significant shift as white marble replaced the
previously used red sandstone as the primary building material. Itmad-ud-Daulah's
tomb became the first Mughal creation entirely crafted from white marble,
.
IASBABA’S PRELIMS TEST SERIES (AIPTS) 2024 – TEST 7 (ART & CULTURE) SOLUTIONS

showcasing exceptional pietra-dura works. Additionally, Jahangir's son Shah Jahan


constructed Jahangir's tomb in Lahore. (Hence statement 1 is correct)
• However, during Aurangzeb's rule, Mughal architecture witnessed a decline.
Aurangzeb, being a puritan, showed little interest in art and architecture. Nonetheless,
he commissioned the construction of Bibi Ka Maqbara in Aurangabad, as a tribute to
his wife Begum Rabia-ud-Daurani. Regrettably, it is often regarded as a poor imitation
of the Taj Mahal. (Hence statement 3 is correct)

Q.22) Solution (d)

Explanation:

Portuguese architectural influence

The arrival of the Portuguese introduced the Iberian architectural style to the regions they
explored. Initially, they constructed trading posts and warehouses, which were later
transformed into fortified towns along the coastlines. (Hence statement 1 is correct)

They also brought with them the concept of 'patio houses' and the Baroque style that
emerged in late 16th-century Europe, emphasizing the power of the Church. This style
featured intricate, theatrical designs aimed at creating a dramatic impact, often incorporating
contrasting colors. (Hence statement 2 is correct)

Here are some notable examples of Portuguese architectural influence:

• The Se Cathedral in Goa, completed in 1619 AD, showcases the Portuguese late-Gothic
style and houses the renowned 'Golden Bell'.
• The Basilica of Bom Jesus in Goa, a World Heritage Site completed in 1605 AD,
exemplifies the Baroque style and is home to the revered body of St. Francis Xavier.
• Castella de Aguada in Mumbai
• St. Paul's Church in Diu, completed in 1610 AD, reflects the Baroque style. (Hence
statement 3 is correct)
• Diu Fort established in 1535 AD on Diu Island's coastline, features a lighthouse and
fortified walls with cannons. The fort complex includes three churches: St. Thomas
Church, St. Paul's Church, and the Church of St. Francis of Assisi.
• The Church of St. Anne in Talaulim, Goa, completed in 1695 AD, is a fine example of
Baroque architecture.
.
IASBABA’S PRELIMS TEST SERIES (AIPTS) 2024 – TEST 7 (ART & CULTURE) SOLUTIONS

Q.23) Solution (c)

Explanation:
Basis Ilberian Architecture Gothic Architecture
Material The Portuguese primarily used brick as Red sandstone and coarse
Used their main construction material, while limestone were the
wood was commonly utilized for roofs predominant materials
and stairwells. (Hence statement 1 is commonly used in their
correct) constructions.
Structural The Portuguese adhered to their The emergence of the Indo-
Variations Western customs and did not introduce Gothic architectural style
any significant structural alterations. can be attributed to the
(Hence statement 2 is correct) British adoption of Indian
elements and forms

Q.24) Solution (a)

Explanation:

Chandraketugarh, situated near the Bidyadhari River in the North 24 Parganas district of West
Bengal, is an intriguing archaeological site. It has gained significant attention due to its
remarkable terracotta sculptures. The excavations at Chandraketugarh have revealed relics
and sculptures from the Maurya, Kushana, and Gupta periods, including artifacts such as
Northern Black Polished Ware (NBPW), punch-marked coins, and gold coins. The terracotta
art found at the site showcases an exceptional level of precision and craftsmanship. (Hence
option a) is correct)

Q.25) Solution (c)

Explanation:

Neo-Classical style

After 1911, the British Raj pursued construction projects in either the Neo-Roman or Neo-
Classical style. Notable examples of this style can be found in the architecture of New Delhi,
executed by Edwin Lutyens and Herbert Baker, which is often likened to the 'Rome of
Hindustan'. (Hence statement 1 is correct)

The characteristics of this phase include:

• Anonymous constructions lacking distinct features


• Congestion and limited artistic expression due to the amalgamation of various
architectural styles
.
IASBABA’S PRELIMS TEST SERIES (AIPTS) 2024 – TEST 7 (ART & CULTURE) SOLUTIONS

• Compromised simplicity, modernity, and utility due to the hybrid nature of the
structures
• Emphasis on circular buildings
• Excessive use of oriental motifs in the execution of Western architectural designs
• Introduction of the concept of upturned domes, exemplified in structures such as the
Supreme Court and Rashtrapati Bhavan(Hence statement 1 is correct)

Q.26) Solution (d)

Explanation:

• The Edicts of Ashoka comprise approximately 33 inscriptions found on pillars,


boulders, and cave walls, which were created by Emperor Ashoka of the Mauryan
Empire between 268 BC and 232 BC. These inscriptions were scattered across the
country and provide the earliest tangible evidence of Buddhism. (Hence statement 1
is correct)
• In these edicts, Ashoka elaborately describes his perspectives on Dhamma, aiming to
address the profound challenges faced by the diverse society. The decipherment of
Emperor Ashoka's edicts was first accomplished by James Prinsep.(Hence statement 2
is correct)
• The Ashokan Edicts are divided into the following:
o Minor Rock Edicts (3)
o Major Rock Edicts (14 edicts and 2 separate ones found in Odisha)
o Minor Pillar Edicts (5)
o Major Pillar Edicts (6 and in one case 7)
• Kandahar Bilingual Rock Inscription, in Greek and in Aramaic, written in the 10th year
of his reign (260 BC), is the first known inscription of Ashoka. (Hence statement 3 is
correct)

Q.27) Solution (b)

Explanation:

Besnagar Pillar Inscription of Heliodorus (Madhya Pradesh)

• The 2nd century BC pillar inscriptions of Heliodorus (the Greek ambassador) belongs
to the Shunga period and was inscribed on a pillar at Besnagar, the site of ancient
Vidisha in Madhya Pradesh. The inscription was written in the Prakrit language and in
Brahmi script (Hence statement 1 is incorrect)
• Heliodorus was an ambassador of Antialcidas, the Greek king of Taxila (now in
Pakistan), in the court of the local Shunga prince Bhagabhadra, in Vidisha.
.
IASBABA’S PRELIMS TEST SERIES (AIPTS) 2024 – TEST 7 (ART & CULTURE) SOLUTIONS

• The column was set up as a Garuda Pillar in honour of God Vasudeva (Vishnu) by
Heliodorus
• In the six-line inscription on the pillar, Heliodorus describes himself as Bhagavata (i.e.
worshipper of God Vasudeva Krishna) indicating that he evidently adopted Hinduism.
(Hence statement 3 is correct)
• The other side of the pillar has a short inscription describing three steps to immortality
(control, generosity and attention) that, when correctly followed, lead to heaven.
• The pillar is a witness to the development of Indo-Greek diplomatic relations, an
indicator of the popularity of the cult of Vasudeva under Vaishnavism and point to
flourishing trade in the peninsular region. (Hence statement 2 is correct)
• The pillar was discovered in 1877 by Alexander Cunningham.
• The pillar is locally called Khamba Baba and is worshipped especially by fishermen.

Q.28) Solution (a)

Explanation:

• The Hathigumpha Inscription, also known as the Elephant Cave Inscription, is located
in the Udayagiri-Khandagiri Caves of Odisha.
• It is considered to be the earliest epigraphic evidence of image worship in Jainism.
(Hence statement 2 is incorrect)
• The inscription was made by King Kharavela and is believed to date back to the 1st
century BC.
• Written in the Prakrit language and Brahmi script, the Hathigumpha Inscription
consists of 17 lines. (Hence statement 3 is correct)
• It serves as the main source of information about Kharavela, the ruler of Kalinga.
• The Hathigumpha Inscription provides a detailed account of Kharavela's reign as a king,
conqueror, patron of culture, and supporter of Jainism. (Hence statement 1 is correct)
• It emphasizes Kharavela's respect for all religions and his patronage of temples.
• The inscription concludes with praises for the king.
• Similar to the Allahabad Prashasti for Samudragupta, the Hathigumpha Inscription is a
prashasti, a poetic composition in praise of the ruler.

Q.29) Solution (b)

Explanation:

Halmidi Inscription (Karnataka)


.
IASBABA’S PRELIMS TEST SERIES (AIPTS) 2024 – TEST 7 (ART & CULTURE) SOLUTIONS

• It is the oldest known Kannada language inscription in the Kadamba script dated
around 450- 500 AD. It is the earliest evidence of the usage of Kannada as an
administrative language. (Hence statement 1 is incorrect)

Aihole Inscription (Karnataka)

• Aihole was the first capital of the Chalukyas. Many inscriptions were found at Aihole,
but the inscription found near Meguti Jain Temple, popularly known as Aihole
inscription or Aihole Prashasti, details many historical events of the Chalukyas. The
Sanskrit inscription is written in Kannada script.
• There is mention about the defeat of Harshavardhana by Pulakeshin II and the victory
of the Chalukyas, over the Pallavas. It also mentions about the shifting of the capital
from Aihole to Badami. (Hence statement 2 is correct)
• The lines were written by Ravikirti, the court poet of Pulakeshin II who reigned from
610 to 642 AD. (Hence statement 3 is correct)

Q.30) Solution (c)

Explanation:

Bhitari Pillar Inscription of Skandagupta (Uttar Pradesh):

• Dated around 455-467 AD, the inscription is extremely important to define the
chronology of the various Gupta rulers and conflict between Skandagupta and the
Pushyamitras and Hunas. (Hence option c) is the correct answer)
• The Bhitari Pillar Inscription is an ancient inscription found in Bhitari, Uttar Pradesh,
India. It is written in the Brahmi script and dates back to the 5th century CE. The
inscription provides valuable information about the reign of Skandagupta, the Gupta
Empire ruler. It mentions Skandagupta's victories over his enemies and his patronage
of various religious and cultural activities. The Bhitari Pillar Inscription is considered an
important historical document that sheds light on the Gupta Empire's political and
cultural achievements during that period.

Q.31) Solution (a)

Explanation:

• In India, the origins of painting can be traced back to ancient rock paintings found in
Bhimbetaka, Mirzapur, and Panchmarhi. Following that, the Indus Valley civilization
introduced painted pottery. However, the true development of the art of painting
began during the Gupta Age, specifically in the 3rd century AD.
.
IASBABA’S PRELIMS TEST SERIES (AIPTS) 2024 – TEST 7 (ART & CULTURE) SOLUTIONS

• Vatsyayana in his book Kamasutra enumerated the six main principles/limbs or


Shadanga of Indian art. They are as follows:
o Roopbheda - Looks and appearance
o Pramana - Proportion, measurement and structure of the object or
subject
o Bhava - Expression
o Lavanya Yojana - Aesthetic composition
o Sadrishya - Resemblance
o Varnikabhanga- Use of brush and colours
• The craft of painting is mentioned in various Brahmanical and Buddhist literature,
where different forms of artistic representation are described.
• One such form is known as Lepya Chitra, which involves depicting myths and legends
on textiles. Another mentioned art form is Lekhya Chitra, which focuses on line
drawings and sketches. (Hence pair 1 is correctly matched)
• Additional types include Dhuli Chitra, which involves painting or drawing on the floor
(Hence pair 2 is incorrectly matched)
• Pata Chitra, which involves painting on cloth or palm leaves. These references
highlight the diverse range of painting styles and mediums present in ancient Indian
art. (Hence pair 3 is incorrectly matched)

Q.32) Solution (d)

Explanation:

Chalcolithic Period paintings

• The use of green and yellow colors was


prevalent in the paintings. (Hence
statement 1 is correct)
• Many of the paintings depicted battle
scenes with men riding horses and
elephants. (Hence statement 2 is correct)
• Some figures in the paintings were shown
carrying bows and arrows, indicating
preparedness for skirmishes.
• The presence of paintings and writings in
the Ashokan and Gupta Brahmi scripts confirmed the existence of human habitation
in these cave sites during the late historical periods.
• Paintings found in Narsinghgarh, Madhya Pradesh, from the same period depicted
spotted deer skins left for drying, suggesting that the art of tanning skins for shelter
and clothing had been perfected by humans.
.
IASBABA’S PRELIMS TEST SERIES (AIPTS) 2024 – TEST 7 (ART & CULTURE) SOLUTIONS

• Musical instruments like the harp were depicted in some of the paintings, along with
complex geometric shapes such as spirals, rhomboids, and circles. (Hence statement
3 is correct)
• The Jogimara Cave site in the Ramgarh Hills, Surguja district, Chhattisgarh, contained
paintings dating back to around 1000 BC.
• Chhattisgarh's Kanker district housed various caves, including Udkuda, Garagodi,
Khairkheda, Gotitola, and Kulgaon, which displayed human figurines, animals, palm
prints, and bullock carts, indicating a more advanced and settled way of life.
• Similar paintings could also be found in the Ghodsar and Kohabaur rock art sites in
Koriya district, Chhattisgarh.
• Chitwa Dongri in Durg district featured interesting findings such as a Chinese figure
riding a donkey, dragon pictures, and agricultural scenes.

Q.33) Solution (c)

Explanation:

Bhimbetka Cave paintings

• The Bhimbetka Cave site is


situated in the southern part of Bhopal,
within the Vindhya Range of Madhya
Pradesh.
• More than 500 caves in
Bhimbetka contain rock paintings, and it
was designated as a UNESCO World
Heritage Site in 2003. (Hence statement
2 is correct)
• The discovery of these caves is credited to archaeologist V.S. Wakankar.
• The oldest paintings in Bhimbetka are estimated to be around 30,000 years old, and
their preservation is due to their location deep inside the caves.
• There is evidence of continuous human occupation of the caves from 100,000 BC to
1000 AD, with many paintings layered on top of each other.
• The paintings in Bhimbetka span different periods including the Upper Palaeolithic,
Mesolithic, Chalcolithic, early historic and medieval ages, although the majority
belong to the Mesolithic era. (Hence statement 3 is incorrect)
• The paintings primarily depict the daily life of prehistoric humans, often represented
as stick-like figures. (Hence statement 1 is correct)
• Various animals such as elephants, bison, deer, peacocks, and snakes are portrayed,
along with weapons like bows, arrows, spears, shields, and swords.
.
IASBABA’S PRELIMS TEST SERIES (AIPTS) 2024 – TEST 7 (ART & CULTURE) SOLUTIONS

• Some paintings feature simple geometric designs and symbols, while other common
themes include dancing, music-playing, and animal fights.
• The paintings provide insights into social life, with depictions of children playing,
women preparing food, and communal dancing.
• A variety of colors, including red ochre, purple, brown, white, yellow, and green, were
used in the paintings, obtained from natural resources. For example, haematite ores
were used to create the color red.
• Similar prehistoric paintings can be found in other nearby locations such as Bellary,
Pachmarhi, Rajgarh, and Tenmalai, which are extensions of the Vindhyan Range into
Uttar Pradesh.
• Limdariha in Bastar district and Oogdi and Sita Lekhani in Surguja district, Chhattisgarh,
were locations where several intriguing rock paintings were discovered.
• In Odisha, the Gudahandi Rock Shelter and Yogimatha Rock Shelter were notable
examples of early cave paintings.

Q.34) Solution (a)

Explanation:

The Apahhramsa School of Art

• The Apahhramsa School of Art originated in Gujarat and the Mewar region of
Rajasthan.
• It was the dominant school of painting in Western India from the 11th to 15th
centuries. (Hence statement 1 is correct)
• Initially, the paintings mainly featured Jain iconography, but later the Vaishnava
School incorporated similar themes. (Hence statement 2 is incorrect)
• The artists introduced the concepts of Gita Govinda (a poem) and secular love into the
paintings.
• Palm leaf was initially used as the medium for painting, but later they transitioned to
paper. (Hence statement 3 is correct)
• Symbolic colors were used, with red, yellow, and ochre being common, while brighter
colors and gold were experimented with in the later phase.
• The human figures depicted in the paintings had distinctive features such as fish-
shaped bulging eyes, a pointed nose, and a double chin.
• The artists aimed to create angular faces in the third and fourth profiles, setting a
trend.
• The figures appeared stiff, and intricate ornamentation was meticulously executed.
• Female figurines had emphasized hips and breasts.
• Animal and bird figurines in the paintings were depicted as toys.
.
IASBABA’S PRELIMS TEST SERIES (AIPTS) 2024 – TEST 7 (ART & CULTURE) SOLUTIONS

• One of the most famous examples of the Apahhramsa School is the Kalpasutra and the
Kalakacharya Katha from the 15th century.

Q.35) Solution (c)

Explanation:

Contributions of Akbar

• Akbar established a dedicated department for illustrating and scribing his documents.
• He created an official artistic studio called Tasvir Khana, where artists were employed
and given the opportunity to develop their individual styles. (Hence statement 2 is
incorrect)
• Akbar viewed paintings as a means of both study and entertainment.
• Recognizing the talent of skilled Indian artists who had previously worked for other
rulers, Akbar invited them to work in his Tasvir Khana, leading to the emergence of
Indian influences in Mughal paintings.
• The paintings of Akbar's era were characterized by the use of three-dimensional
figures and continued utilization of foreshortening techniques. (Hence statement 1 is
correct)
• Calligraphy was encouraged and incorporated into the paintings during this period.
(Hence statement 3 is correct)
• The focus of the artists shifted from popular art to court art, with a greater emphasis
on depicting scenes from court life rather than the lives of the common people.
• Notable painters of this period include Abdus Samad, Mir Sayyid Ali, Daswanth,
Basawan, and Kesu Das.
• Prominent illustrated manuscripts created during Akbar's reign include Tutinama,
Razmnama, Baburnama, Hamzanama, Tarikh-i-Alfi, Anvar-i-Suhaili, and Gulistan of
Sadi.

Q.36) Solution (c)

Explanation:

Differences between Rajput Style and Mughal Style of Paintings


.
IASBABA’S PRELIMS TEST SERIES (AIPTS) 2024 – TEST 7 (ART & CULTURE) SOLUTIONS

Q.37) Solution (b)

Explanation:

Points of Rajput Style Mughal Style


Difference
Type It was initially based on mural and It is based on the Persian miniature
fresco forms. In the later period, painting style.
the miniature painting form (Hence statement 1 is correct)
became dominant.
Themes It is usually devotional or religious It usually depicts the Mughal emperor
in nature. and his household. Pomp and show,
(Hence statement 2 is correct) battles and hunting scenes are also very
popular themes.
Peculiarity It uses Hindu symbols like the lotus, They focus is either on the person in the
peacock and swan very frequently. picture or on trees, camels and falcons.
(Hence statement 3 is incorrect)
Time 17th-18th centuries was the major This style flourished from the 16th- 18th
period period. centuries.
Pattachitra paintings

• Pattachitra is a traditional painting style from Odisha, derived from the Sanskrit words
"patta" meaning canvas/cloth, and "chitra" meaning picture. These paintings combine
classical and folk elements, with a greater inclination towards the latter.
• The artwork is created on treated cloth, using colors sourced from natural substances
like burnt coconut shells, hingula, ramaraja, and lamp black. Pencil or charcoal is not
utilized; instead, the outlines are drawn with a brush in red or yellow, followed by filling
in the colors. The background is adorned with foliage and flowers, and the paintings
feature intricately crafted frames. A coat of lacquer is applied to give the artwork a
glossy finish.
• The themes of Pattachitra paintings draw inspiration from the Jagannath and
Vaishnava sects, and occasionally from the Shakti and Shaiva sects as well. Raghurajpur
in Odisha is renowned for this art form. The imagery in Pattachitra paintings resembles
the ancient murals found in the state, particularly those in Puri and Konark. When
created on palm leaf, Pattachitra is known as Talapattachitra.
.
IASBABA’S PRELIMS TEST SERIES (AIPTS) 2024 – TEST 7 (ART & CULTURE) SOLUTIONS

Q.38) Solution (a)

Explanation:

Famous Metal Crafts in India

Name of the Metal Produced Mainly Details


Craft in
Bidri Craft (GI) Karnataka Blackened zinc and copper alloy is inlaid in
(Hence pair 1 is thin sheets of silver to make items such as
incorrectly hookahs, vases, earrings, decorative items,
matched) etc. which also have a good export market.

Aranmula Kannadi Kerala Handmade metal-alloy mirrors, made in


(MetalMirror) (GI) (Hence pair 2 is Aranmula, a small town in Kerala, are rare
incorrectly pieces of craftwork. The alloy is polished for
matched) several days to obtain a mirror-like reflective
surface. The composition of the alloy is a trade
secret of the families involved.

Nettur Petti jewellery Kerala This is an antique handicraft usually made


boxes using rosewood and brass curios. It has
intricate patterns which are inspired by the
temple architecture and traditional art forms
of Kerala.

Chandi Tarkashi Odisha An alloy of 90% or more pure silver is used for
(Silver Filigree) (Hence pair 3 is techniques such as granulation, snow glazing
correctly and casting to make innovative ornaments.
matched)
Swamimalai Bronze Tamil Nadu Swamimalai is a traditional site where this
Icons (GI) craft has been practiced since the Chola
period. Bronze and 'panchaloha' (copper,
brass, lead silver and gold) idols are made for
religious purposes

Q.39) Solution (d)

Explanation:

Buddhist Monuments at Sanchi (Madhya Pradesh):


.
IASBABA’S PRELIMS TEST SERIES (AIPTS) 2024 – TEST 7 (ART & CULTURE) SOLUTIONS

• The monuments at Sanchi encompass a series of Buddhist structures that originated


during different historical periods, including the Mauryan Empire (3rd century BC),
Gupta Empire (5th century AD), and ending around the 12th century AD.
• India's most well-preserved group of Buddhist monuments is situated on a flat-topped
sandstone hill, which towers approximately 300 feet (90 meters) above the
surrounding landscape. Recognized as a UNESCO World Heritage site in 1989, these
monuments hold significant historical and cultural importance.(Hence statement 1 is
correct)
• Sanchi is recognized as the oldest surviving Buddhist sanctuary in the world. (Hence
statement 2 is correct)
• These Buddhist monuments consist of monolithic pillars, palaces, temples, and
monasteries.
• The notable monuments at Sanchi include the Great Sanchi Stupa, Ashokan Pillar,
Shunga Pillar, Schism Edicts, Siri Satakarni inscriptions from the Satavahana period, and
various other stupas.
• The relief sculptures present at Sanchi depict various aspects of Buddhism, but they
do not feature the image of Buddha. (Hence statement 3 is correct)
• The Toranas (gateways to the stupas) at Sanchi also showcase sculptures of Yakshi and
Yaksha.
• The image of the Sanchi Stupa is depicted on the reverse side of the Indian Rs 200
currency note.

Q.40) Solution (c)

Explanation:

Toy Making

• Toy making is both an art form and a thriving business.


• The earliest evidence of toy carts can be traced back to the Harappan civilization and
continued into the early historic period.
• Archaeological sites like Alamgirpur, Ambkheri, Kalibangan, and Lothal have yielded
copper, bronze, and terracotta toy carts from the Harappan period.
• Similar cart models have been discovered in early historic sites such as Atranjikhera,
Nagarjunakonda, Sambhar, Sonapur, Ujjain, and Brahmpuri.
• Toys in ancient and medieval times were made from various materials such as clay,
paper, papier-mâché, and painted or lacquered wood.
• Redwood was used to create beautiful dolls, like the Tirupati dolls popular in Andhra
Pradesh.
• Rajasthan is renowned for dolls and stuffed toys crafted from vibrant cloth.
• Assam's traditional dolls are made from pith or Indian cork.
.
IASBABA’S PRELIMS TEST SERIES (AIPTS) 2024 – TEST 7 (ART & CULTURE) SOLUTIONS

• Southern Indian states specialize in making toys from different types of wood.
• Mysore and Channapatna in Karnataka are known for unique toys made from
lacquered wood, considered as crafts due to their exquisite appearance.
• In Kondapalli, Andhra Pradesh, the local soft wood known as "ponki" is used to create
a popular toy called the Ambari Hathi.
• Channapatna Toys (Karnataka): Traditionally made from ivory wood. As a result of its
popularity, Channapatna town is also known as Gombegala Ooru (Toy Town). (Hence
pair 1 is correctly matched)
• Kondapalli Bommalu (Andhra Pradesh): This is made using a soft wood. The wooden
piece is heated to remove any moisture, and subsequently, different parts of the toy
are carved separately and then glued together. An example is Ambari Hathi. (Hence
pair 2 is correctly matched)
• Kinnal Toys (Karnataka): This unique wooden craft flourished under the royal
patronage of the Vijayanagara Empire. (Hence pair 3 is incorrectly matched)

Q.41) Solution (c)

Explanation:

Raga

• The term 'raga' originates from the Sanskrit word 'Ranj,' which means to bring delight
and satisfaction. Ragas are the foundation of melody, while tala serves as the basis for
rhythm. Each melodic structure of a raga possesses a distinct personality and evokes a
particular mood through its sounds. The fundamental component essential for a raga
is the tonic note upon which it is built. Based on the number of notes present, ragas
can be categorized into three main jaatis or categories:
o Audav/Odava Raga: This pentatonic raga consists of five notes.
o Shadava Raga: This hexatonic raga contains six notes.
o Sampurna Raga: This heptatonic raga comprises seven notes.
• A raga is not merely a scale or mode but rather a scientific, precise, subtle, and
aesthetically pleasing melodic form with its unique ascending and descending
movements. It encompasses a full octave or a series of five, six, or seven notes. There
are three major types of Raga or Raga Bhed:
o Shuddha Raag: In this raga, the nature and form remain unchanged even if
additional notes not present in its composition are played. (Hence pair 1 is
incorrectly matched)
o Chhayalag Raag: The nature and form of this raga alter when notes not
originally present in its composition are played. (Hence pair 2 is incorrectly
matched)
.
IASBABA’S PRELIMS TEST SERIES (AIPTS) 2024 – TEST 7 (ART & CULTURE) SOLUTIONS

o Sankeerna Raag: This raga combines two or more ragas. (Hence pair 3 is
correctly matched)
• Therefore, every raga should contain the basic five notes. Within these ragas:
• The 'King' or 'Vaadi' is the principal note upon which the raga is constructed and is
used most frequently in compositions.
• The 'Queen' or 'Samvaadi' corresponds to the fourth or fifth note in relation to the
principal raga and holds the second most significant position.
• All notes in the composition, except the vaadi and samvaadi, are known as anuvaadi.
• The notes that are not present in the composition are referred to as vivadi.
• Furthermore, the ascent of notes implies that each note is higher than the preceding
one, following the pattern sa re ga ma pa dha ni. This ascent is known as Aroha.
Similarly, the descent of notes is called Avaroha, where each note is lower than the
preceding ones, such as ni, dha, pa, ma, ga, re, sa. Depending on the ascent and
descent of notes, ragas can be classified into three speeds or Laya: Vilambit (slow),
Madhya (medium), and Drut (fast).
• Although Carnatic music encompasses 72 melas or parent scales upon which ragas are
based, Hindustani music repertoire primarily focuses on six main ragas. Each of these
ragas is associated with specific times and seasons and evokes particular emotions
(Rasa).

Q.42) Solution (b)

Explanation:

• Music holds a profound significance in every culture, and India boasts a rich heritage
of musical brilliance. Legend has it that the sage Narada Muni introduced the art of
music to Earth, along with the knowledge of Nada Brahma, the universal sound.
Archaeological discoveries, such as the seven-holed flute from the Indus Valley
Civilisation and the ancient Ravanahatha instrument from the Hela Civilisation in Sri
Lanka, provide evidence of music's early existence. (Hence statement 1 is incorrect)
• Literary references to music can be traced back approximately 2000 years to the Vedic
era. The descending order of all seven notes of the Kharaharapriya raga can be found
in the Sama Veda. The science of music, known as Gandharva Veda, is an Upaveda of
the Sama Veda. (Hence statement 2 is correct)
• The Aitareya Aranyaka mentions parts of the Veena instrument, while the Jaiminiya
Brahmana discusses dance and music together. Musicologists have put forth theories
regarding the word 'Om' being the source of all ragas and notes. Panini, in the 4th
century BC, made the earliest proper reference to the art of music, while Bharata's
Natyashastra, compiled between 200 BC and 200 AD, discussed musical theory.
• The Khyal style, derived from the Persian word meaning 'idea or imagination,' is
credited to Amir Khusrau and is widely popular among artists due to its
.
IASBABA’S PRELIMS TEST SERIES (AIPTS) 2024 – TEST 7 (ART & CULTURE) SOLUTIONS

improvisational possibilities. Khyal compositions, also known as Bandish, consist of


short songs ranging from two to eight lines. (Hence statement 3 is correct)
• Hussain Shah, a ruler of the Jaunpur Sultanate in the 15th century, provided significant
patronage to Khyal. A notable feature of Khyal is the frequent use of taans within the
composition, giving it a distinct identity. As a result, Khyal music allocates less space
for Alap compared to Dhrupad.

Q.43) Solution (c)

Explanation:

Dhrupad Style

• Dhrupad is one of the oldest and most prominent forms of Hindustani classical music,
mentioned in Natyashastra (200 BC-200 AD). Its roots can be traced back to older
forms like Prabhanda and Dhruvapada.(Hence statement 1 is incorrect)
• Dhrupad derives its name from 'dhruva' and 'pada,' denoting both the verse form and
singing style. It gained prominence in the 13th century and flourished during Emperor
Akbar's reign. (Hence statement 2 is correct)
• Akbar employed and patronised musical masters including Swami Haridas and Mian
Tansen. Tansen (one of the Navaratna or nine gems in Akbar's court) and Baiju Bawra
were the students of Swami Haridas and were major proponents of dhrupad music
during the 15th-16th centuries. Dhrupad became the major form of singing in the
medieval period but fell into decline in the 18th century.
• Dhrupad is essentially a poetic form that is incorporated into an extended presentation
style that is marked by precise and overt elaboration of a raga.
• Dhrupad features an extended presentation style with precise elaboration of a raga. It
begins with Alap, a wordless rendition that gradually builds tempo, setting the mood
of the raga. (Hence statement 3 is correct)
• Dhrupad compositions use Sanskrit syllables and have four to five stanzas. It is typically
performed by a duo of male vocalists accompanied by Tanpura and Pakhawaj.
• Dhrupad encompasses different banis or styles, including Dagari Gharana (known for
emphasis on alap), Darbhanga Gharana (balancing alap and bandish), Bettiah Gharana
(giving prominence to bandish), and Talwandi Gharana (based in Pakistan).
• Prominent exponents include the Gundecha Brothers from Dagari Gharana, Ram
Chatur Mallick and Pandit Siyaram Tewari from Darbhanga Gharana, the Mishra family
from Bettiah Gharana, and Talwandi Gharana based in Pakistan.
.
IASBABA’S PRELIMS TEST SERIES (AIPTS) 2024 – TEST 7 (ART & CULTURE) SOLUTIONS

Q.44) Solution (c)

Explanation:

• Thumri, a semi-classical form of Indian music originating in Uttar Pradesh, is known


for its mixed simple ragas and versatile compositions. (Hence statement 1 is correct)
• It is inspired by the Bhakti movement, with a focus on romantic or devotional themes,
often centered on a girl's love for Krishna. The language of the compositions is
typically Hindi in the Braj Bhasha dialect. (Hence statements 2 and 3 are incorrect)
• Thumri is primarily sung by female vocalists and is characterized by its inherent
sensuality, allowing for improvisation and greater flexibility with the use of raga. It
encompasses lighter forms such as Dadra, Hori, Kajari, Saavan, Jhoola, and Chaiti.
(Hence statement 4 is correct)
• Thumri shares a connection with the classical dance form Kathak. The prominent
Gharanas (schools) of thumri are based in Varanasi and Lucknow, and renowned artists
like Begum Akhtar, Girija Devi, and Chhannulal Mishra have left an enduring legacy in
the realm of thumri with their captivating voices and impeccable singing skills.

Q.45) Solution (b)

Explanation:

Purandara dasa is referred to as the father of Carnatic music but all other are considered as
Trinity of Carnatic music.

Purandara Dasa (1484-1564)

• He was one of the founding proponents of Carnatic music.


• He was a devotee of Lord Krishna.
• He is widely referred to as the 'Pitamaha or father/grandfather of Carnatic Music'.
• He is believed to be an avatar or incarnation of the sage Narada.
• His most famous composition includes Dasa Sahithya.

TRINITY OF CARNATIC MUSIC

Thyagarajan (1767-1847)

• Most compositions in Telugu and few in Sanskrit and in praise of Lord Rama.
• His very famous composition is Pancharatna Kritis.
• A crater on planet Mercury is named Thyagaraja.
• Thyagaraja created several new ragas

Muthuswamy Dikshitar (1775-1835)

• Most compositions are in Sanskrit and relate to Hindu gods and temples.
.
IASBABA’S PRELIMS TEST SERIES (AIPTS) 2024 – TEST 7 (ART & CULTURE) SOLUTIONS

• He emphasised the use of Gamaka (ornamentation).


• His signature name was Guruguha which is also his mudra.
• He was also a Veena player.

Shyama Sastri (1762-1827)

• Older than the other two. Compositions mainly in Telugu in praise of Goddess
Kamakshi.
• He composed less kritis as compared to the other two.
• His signature or mudra was Syama Krishna.
• The Svarajati musical genre can be attributed to him.

Q.46) Solution (a)

Explanation:

Baul:

• The Bauls are not only a type of music but also a Bengali religious sect. Baul Sangeet,
their music, is a particular type of folk song influenced by the Hindu Bhakti movements
and Sufi songs. It represents a long heritage of preaching mysticism through songs in
West Bengal, Assam, and Tripura. The prominent exponents of Baul music include Yotin
Das, Purno Chandra Das, Lalon Fakir, Naboni Das, and Sanatan Das Thakur Baul.

Wanawan:

• Wanawan is the folk music from Jammu and Kashmir, commonly sung during wedding
ceremonies and considered auspicious.

Pandavani:

• Pandavani is a type of folk music based on the grand epic Mahabharata, with Bhima as
the hero. It involves singing (gayan) and playing an instrument (vadan), often
accompanied by the rhythm of a tambura. Teejanbai from Chhattisgarh is a well-known
artist in this musical field.

Panihari:

• Panihari is a musical form from the state of Rajasthan, thematically related to water.
The songs typically depict women fetching water from a nearby well and carrying it
back to their households in matkas over their heads. They often describe the scarcity
of water and the daily concerns of village women clustered around the well.

Lavani:
.
IASBABA’S PRELIMS TEST SERIES (AIPTS) 2024 – TEST 7 (ART & CULTURE) SOLUTIONS

• Lavani is one of the most famous folk dance forms from Maharashtra and a popular
genre of music in the region. It combines traditional dance and song, usually
performed to the beats of the Dholki, a percussion instrument. The music has a
powerful rhythm and beats, perfect for lively dancing.

Dandiya:

• Dandiya Raas is the traditional folk dance form in Gujarat, associated with scenes of
Holi and the lila (divine play) of Krishna and Radha at Vrindavan. It is performed during
Navaratri evenings and involves men and women dancing in two circles with sticks in
their hands. Dandiya Raas and Garba are different forms of Raas, with Dandiya Raas
being the most popular form during Navaratri in Gujarat.

Q.47) Solution (c)

Explanation:

• Distinct dance styles have developed in various parts of India, each with its unique
characteristics. However, all these dance forms adhere to the fundamental principles
outlined in the Natya Shastra, where the primary rule emphasizes the importance of
learning from a guru. The guru imparts the knowledge of different traditions and
sampradayas (lineages) to the disciple. This tradition, known as 'guru-shishya
parampara,' lies at the heart of Indian classical art forms.
• Presently, as per the Sangeet Natak Akademi, there exists eight classical dance forms
in India, which include Bharatanatyam, Kuchipudi, Kathakali, Mohiniyattam, Odissi,
Manipuri, Kathak and Sattriya. (Hence option c) is correct)
• Bihu and Garba are Folk dances of India not considered as Classical dance forms in
India

Q.48) Solution (d)

Explanation:

Bharatanatyam

• Bharatanatyam, the oldest among all classical dance forms, derives its name from
'Bharata Muni' and 'natyam', meaning dance in Tamil. It is also associated with the
concepts of 'Bhava', 'Raga', and 'Taal'.
• The dance form originated from the solo performances of temple dancers or
'devadasis' in Tamil Nadu, known as 'Sadir' or 'Dashiattam'.
• E. Krishna Iyer played significant roles in reviving and popularizing
Bharatanatyam.Rukmini Devi Arundale (1904-86), another famous proponent of
Bharatanatyam, is remembered for giving global recognition to the dance.
.
IASBABA’S PRELIMS TEST SERIES (AIPTS) 2024 – TEST 7 (ART & CULTURE) SOLUTIONS

• The Bharatanatyam recital consists of different elements defined by four dance


teachers of Thanjavur in the 19th century.
• The elements include Alarippu, Jatiswaram, Shabdam, Varnam, Padam, Jawali, and
Thillana, each serving a specific purpose within the performance.
o Alarippu (flowering bud) - It is an invocatory piece of performance which
includes basic dance postures and is accompanied with rhythmic syllables. It
symbolises offering of respects to God, the guru and the audience and to seek
the blessings of God. It is the first dance piece in Bharatanatyam performance.
o Jatiswaram - It is the Nritta component and is devoid of expressions, and it
includes different poses and movements.
o Shabdam-It is the dramatic element with expressed words, which includes the
abhinaya in the song. It is generally in praise of the glory of God.og hod has
hogy hid
o Varnam - It is the Nritya component. It is a combination of dance and emotions,
and is the most important part of the whole performance. It is synchronised
with tala and raag, to express the story.
o Padam- It refers to a mastery over the abhinaya (expression) of the spiritual
message, by the artist. Music becomes light, dance becomes emotional.
o Jawali - These are short love-lyrics performed at a faster tempo.
o Thillana - It is the concluding stage of the performance, and comprises pure
dance (Nritta) with exuberant movement and intricate rhythmic variations.
• Bharatanatyam is often referred to as the 'fire dance' due to its resemblance to the
movements of a dancing flame.
• The dance form emphasizes both the Tandava (vigorous) and Lasya (graceful) aspects,
with a focus on mudras (hand gestures).
• Notable mudras include 'Katakamukha Hasta', symbolizing 'Om', and 'Ekcharya
lasyam' style, where one dancer portrays multiple roles.
• In Bharatanatyam, the dancer maintains bent knees and distributes weight equally
across both feet.
• Renowned proponents of Bharatanatyam include Yamini Krishnamurthy, Lakshmi
Viswanathan, Padma Subramanyam, Mrinalini Sarabhai, Mallika Sarabhai, and Alarmel
Valli.
• (Hence option d) is correct)

Q.49) Solution (a)

Explanation:

• In the temples of Kerala, two dance-drama forms called Ramanattam and Krishnattam
were developed by feudal lords, narrating stories from the Ramayana and
Mahabharata. These folk drama traditions later served as the foundation for Kathakali,
.
IASBABA’S PRELIMS TEST SERIES (AIPTS) 2024 – TEST 7 (ART & CULTURE) SOLUTIONS

a theatrical art form combining music, dance, and drama. Kathakali is closely
associated with Koodiyattam, an ancient Sanskrit drama tradition, and other martial
arts performances.(Hence statement 1 is correct)
• However, with the decline of the feudal system, Kathakali faced a decline as an art
form. It was revived in the 1930s by the renowned Malayali poet V. N. Menon, under
the patronage of Mukunda Raja.

Key features of Kathakali dance include:

• Kathakali is predominantly performed by all-male troupes.(Hence statement 2 is


incorrect)
• The use of props in Kathakali is minimal, while elaborate facial makeup and headgear
are employed to portray different characters. Each color used in the makeup holds its
own significance.
• Kathakali combines both dance and drama, with a seamless integration of the two.
• Many Kathakali performances revolve around the eternal conflict between good and
evil, drawing inspiration from stories found in epics and puranas. It is often referred to
as the "Ballad of the East."
• The language used in Kathakali songs is Manipravalam, a blend of Malayalam and
Sanskrit.
• Music plays a crucial role in conveying the essence of the drama, with various
compositions used during the performance to enhance its depth.
• Hand gestures, facial expressions, and eye movements are integral to Kathakali, with
intricate movements conveying different emotions. Nine primary facial expressions,
known as "Navarasas," are taught to express various emotions.
• Kathakali is typically performed in open-air theaters or temple premises in Kerala,
with the natural surroundings providing a backdrop. Brass lamps are used for lighting.
(Hence statement 3 is correct)
• The commencement and conclusion of a Kathakali recital are marked by the arrival of
dawn, accompanied by the continuous rhythm of drums such as chenda and maddala.
• Kathakali symbolizes the element of sky or ether.
• Notable exponents of Kathakali include Guru Kunchu Kurup, Gopinath, Kottakal
Sivaraman, Rita Ganguly, and others.

Q.50) Solution (d)

Explanation:

Bhutharadhana
.
IASBABA’S PRELIMS TEST SERIES (AIPTS) 2024 – TEST 7 (ART & CULTURE) SOLUTIONS

• Bhuta, a term referring to spirits, is a traditional practice that involves worshiping


various divine beings such as ancestors, different avatars of Hindu Gods, and local
folklore heroes. (Hence statement 1 is correct)
• This practice is predominantly observed in the coastal regions of Karnataka. (Hence
statement 2 is correct)
• Kola, a ritualistic theatre, is the primary form of Bhuta worship.
• Nowadays, themes from Vaishnavism, Shaktism, and Shaivism are commonly
incorporated into Bhuta worship.
• The performers in Bhuta worship don elaborate headgear and vibrant costumes.
(Hence statement 3 is correct)

Important points

• Bhutaradhana, also known as Bhootaradhane, is a traditional ritualistic performance


prevalent in certain regions of Karnataka, particularly in the coastal districts. It is a form
of spirit worship that involves invoking and appeasing the spirits of ancestors and local
deities.
• During Bhutaradhana, the performers, known as Bhoota Kola artists or Bhoota sevaks,
dress up in elaborate costumes and colorful masks representing various deities and
spirits. They undergo a ritualistic possession, where they believe that the spirits enter
their bodies and communicate with the human world.
• The performances are accompanied by rhythmic music and chants, creating a mystical
atmosphere. The rituals and gestures performed by the Bhoota sevaks are believed to
bring blessings, protection, and healing to the community.
• Bhutaradhana holds significant cultural and religious importance in the local
communities, and it is often performed during religious festivals, village gatherings,
and special occasions. It is a unique blend of mythology, folklore, and spiritual beliefs,
showcasing the rich cultural heritage of Karnataka.
• In recent years, Bhutaradhana has gained attention as a form of cultural tourism,
attracting visitors who are intrigued by its vibrant performances and spiritual
significance. Efforts are being made to preserve and promote this traditional art form,
ensuring its continuity for future generations.

Q.51) Solution (a)

Explanation:

Rathwa ni Gher (Gujarat)

• Rathwa ni Gher, a tribal dance of the Rathwa tribal community in Gujarat, showcases
its ancient and sophisticated nature through various elements. This dance form is
typically performed during the Kavant Festival, which coincides with Holi. The dancers
.
IASBABA’S PRELIMS TEST SERIES (AIPTS) 2024 – TEST 7 (ART & CULTURE) SOLUTIONS

adorn intricate makeup, move in synchronized footsteps, spin vigorously, and produce
a mesmerizing symphony using indigenous musical instruments.

Tippani (Gujarat)

• Tippani, originating from the Saurashtra region of Gujarat, is a dance exclusively


performed by women. In this dance, the women rhythmically beat the floor with two
long wooden sticks joined by a square, wooden, or iron block, while singing a folk song.

Garba (Gujarat)

• Garba, a well-known folk dance of Gujarat, is performed during Navratri. The term
"Garba" actually refers to the "Garbha deep," an earthen pot with holes in which a
lamp is lit. Women dance in circular movements around the illuminated pot,
accompanied by rhythmic clapping.

Dandiya Raas (Gujarat)

• Dandiya Raas, another lively dance form from Gujarat, involves the use of polished
sticks or dandiyas. This dance represents a mock fight between the goddess Durga and
the demon Mahishasura.

Charba (Himachal Pradesh)

• It is the popular folk dance of Himachal Pradesh, performed during the Dussehra
festivities.

Q.52) Solution (c)

Explanation:

Yakshagana:

• Yakshagana, which is still practiced in Karnataka and parts of Kerala, is believed to be


one of the oldest theatre traditions. Its origins can be traced back to the royal courts
of the Vijayanagara empire and it was performed by a specific community called
Jakkula Varu. Initially, it involved a single artist performing a descriptive dance-drama,
but later evolved into a more elaborate dance drama form. The influence of the
Vaishnava Bhakti Movement is strongly evident in Yakshagana. (Hence option c) is
correct)

Chavittu Natakam:

• Chavittu Natakam is an ancient maritime theatre form from the coastal regions of
Kerala. It is a vibrant Christian art form that incorporates elements from both European
and Kerala art traditions. Its roots can be traced back to the arrival of the Portuguese
.
IASBABA’S PRELIMS TEST SERIES (AIPTS) 2024 – TEST 7 (ART & CULTURE) SOLUTIONS

in the region. The themes of Chavittu Natakam revolve around stories from the Bible
and the heroic exploits of legendary Christian warriors.

Kuruvanji:

• Kuruvanji, which emerged approximately 300 years ago, is characterized by classical


Tamil poetry and songs. The first Kuruvanji composition was created by Tirikooda
Rasappa Kavirayar. The central theme revolves around a love-stricken heroine, and the
term "Kuruvanji" literally translates to "fortune-teller," referring to the character who
predicts the fate of the heroine. It is performed in the form of a dance ballet, with
Bharatanatyam being the primary dance style in Tamil Nadu.

Pragati Veshaalu:

• Pragati Veshaalu is a folk tradition popular in the Telangana region and the Krishna
district of Andhra Pradesh. It primarily involves role-playing, with a central character
known as vesham (disguise) and various supporting characters.

Q.53) Solution (a)

Bhavabhuti

• Uttara Ramacharitam (The later life of Rama). It was written in 700 AD.
Mahaviracharita and Malatimadhava are other two dramas ascribed to him.

Harshavardhana

• Ratnavali(The love story of princess Ratnavali, daughter of the King of Ceylon and King
Udayana. The mention of celebration of Holi for the first time can be found here.)
• Nagananda (Story of how Prince Jimutavahana gives up his own body to stop a sacrifice
of serpents to the divine Garuda. One unique characteristic in this drama is the
invocation to Lord Buddha in the Nandi verse)
• Priyadarsika (union of Udayana and Priyadarsika, daughter of King Dridhavarman)

Bhasa

• Swapnavasavadatta (Vasavadatta in dream) is one of the best dramas written by him.


Many of his plays are based on themes from the two great epics. The plays based on
the Ramayana are Pratima and Abhisheka, while Madhyama- vyayoga,
Dutaghatotkacha, Karnabhara and Urubhanga (story of Duryodhana during and after
his fight with Bhima) are based on the Mahabharata. Pancharatra and Charudatta are
some other works. (Hence pair a) is incorrectly matched)

Kalidasa
.
IASBABA’S PRELIMS TEST SERIES (AIPTS) 2024 – TEST 7 (ART & CULTURE) SOLUTIONS

• Malavikagnimitram (The love story of Malavika a maiden of the Queen and Agnimitra,
the son of Pushyamitra Shunga)
• Vikramorvasiyam (Love story of Vikram and Urvasi)
• Abhigyana Shakuntalam (The recognition of Shakuntala as mentioned in the
Mahabharata)

Sudraka

• Mricchakatika (The Little Clay Cart) is a story of a love affair of a young Brahmin
Charudatta and a wealthy courtesan

Vishakhadatta

• Mudrarakshasa (a political drama and narrates the ascent of King Chandragupta


Maurya to power in India)
• Devi Chandraguptam

Q.54) Solution (d)

Explanation:

Ravanchhaya (Odisha)

• Ravanchhaya, a popular form of shadow puppetry from Odisha, is known for its
captivating and dramatic postures. (Hence statement 1 is correct)
• It utilizes deer skin puppets, which lack articulated joints, adding complexity to the art
form. (Hence statement 2 is correct)
• Non-human characters such as trees and animals are commonly depicted through the
puppets. (Hence statement 3 is correct)
• The skilled Ravanchhaya artists, mainly from the Bhat community, possess exceptional
training in their craft.
• Their performances create a mesmerizing and emotionally evocative theatrical
narrative.

Q.55) Solution (c)

Explanation:

Indian Circus

• The Great Indian Circus was the first modern Indian circus, founded by Prof.
Vishnupant Chatre, an accomplished equestrian and singer. It flourished under the
.
IASBABA’S PRELIMS TEST SERIES (AIPTS) 2024 – TEST 7 (ART & CULTURE) SOLUTIONS

patronage of the Raja of Kurduvadi, under whom Chatre worked as a stable master.
(Hence statement 1 is correct)
• The first performance of the Great Indian Circus was held in 1880 following which it
toured extensively throughout the country as well as on foreign shores such as Ceylon
and South-East Asia.
• Chatre gained appreciation everywhere. However, his tour to North America was a
failure where he could not match the size and grandiose of his competitors.
Vishnupant Chatre is called the father of the Indian circus.

KEELERI KUNHIKANNAN

• Chatre met Keeleri Kunhikannan during his tour in the city of Tellicherry (now
Thalassery) on the Malabar Coast. Keeleri Kunhikannan worked as a martial arts trainer
and gymnastic instructor. On Chatre's insistence, Keeleri started training acrobats at
his academy. In 1901, he opened a circus school at Chirakkara (Kerala), which became
the epicentre of the circus revolution in India. (Hence statement 2 is correct)
• In 1904, one of Kunhikannan's students Pariyali Kannan started his own circus
company by the name of Malabar Grand Circus. Other companies such as Whiteway
Circus (1922), Great Ramayan Circus (1924), the Great Lion Circus, the Eastern Circus
and the Fairy Circus, among others, were also started by the students of Kunhikannan.
Thus, Kerala came to be known as the 'cradle of the Indian circus'. (Hence statement
3 is incorrect)
• Kunhikannan's academy also gave rise to a number of acrobats who gained national
and in- ternational acclaim. Kannan Bombayo, a ropedancer, graduated from
Kunhikannan's acad- emy in 1910, and went on to perform for several European and
American circus companies. After Kunhikannan's death in 1939, his student M. K.
Raman continued his legacy. In 2010, the Government of Kerala started a circus
academy in Thalassery in honour of Keeleri Kunhikannan. He has rightly been
conferred the epithet 'the father of Kerala circus'.

Q.56) Solution (a)

Explanation:

• Kalaripayattu, one of the ancient martial arts in India, originated in the state of Kerala
around the 3rd century BC and is now practiced in various parts of Southern India. The
term "Kalari" in Malayalam refers to a specific type of training hall or gymnasium
where martial arts are taught and practiced. (Hence statement 1 is correct)
• This art form encompasses mock duels involving armed and unarmed combat, as well
as physical exercises. Unlike other martial arts, Kalaripayattu does not involve
drumming or singing, as the focus is primarily on the fighting style. (Hence statement
2 is incorrect)
.
IASBABA’S PRELIMS TEST SERIES (AIPTS) 2024 – TEST 7 (ART & CULTURE) SOLUTIONS

• Footwork is considered crucial in Kalaripayattu, along with kicks, strikes, and training
with weapons. It is worth noting that women also actively participate in this martial
art. Kalaripayattu retains its traditional rituals and ceremonies.
• Kalaripayattu incorporates various techniques and elements, including Uzhichil
(massage with Gingli oil), fighting with Otta (an 'S'-shaped stick), Maipayattu (body
exercises), Koltharipayattu (wooden weapon-based combat), Angathari (sharp metal
weapons and sticks), and Verumkai (bare-handed combat).
• The Sports Ministry recently announced the inclusion of four "indigenous games" in
the upcoming edition of the Khelo India Youth Games, which will be hosted in
Haryana's Panchkula next year. The fourth edition of the Games will feature Manipur's
martial arts game Thang-Ta, Punjab's Gatka, Kerala's Kalaripayattu, and Madhya
Pradesh-Maharashtra's popular Mallakhamba. Additionally, Yogasana has also been
added to the lineup for this edition of the Games. (Hence statement 3 is correct)

Q.57) Solution (d)

Explanation:

Thang-ta and Sarit Sarak

• Thang-ta, an armed martial art, was developed by the Meitei people of Manipur and
is renowned as one of the most deadly combat forms. (Hence statement 1 is correct)
• Sarit Sarak, on the other hand, is an unarmed art form that focuses on hand-to-hand
combat.(Hence statement 2 is correct)
• Both Thang-ta and Sarit Sarak have a history dating back to the 17th century, where
they were effectively employed by Manipuri kings against the British.
• During British rule, these martial art forms were prohibited following the British
capture of the region. (Hence statement 3 is correct)
• However, after India gained independence, there was a revival of interest in these art
forms.
• Thang-ta primarily involves the use of swords (Thang) and spears (Ta), making them
the central elements of this martial art.
• The combined practice of Thang-ta and Sarit Sarak is referred to as Huyen Langlon.
• Thang-ta, known as "The Art of Sword and Spear," is the traditional martial art of
Manipur, a state in northeast India.
• The term Thang refers to the 'sword,' while Ta signifies the 'spear,' making these two
weapons the primary components of Thang-ta.
• Thang-ta combines the use of external weapons such as the sword, spear, and dagger
with internal exercises that focus on physical control through precise movements
synchronized with breathing patterns.
.
IASBABA’S PRELIMS TEST SERIES (AIPTS) 2024 – TEST 7 (ART & CULTURE) SOLUTIONS

• Due to its significance, Thang-ta is only practiced on rare occasions or in exceptional


circumstances, as it is deeply rooted in Manipur's rich heroic history.
• This intricate physical and cultural system involves breathing techniques, meditations,
and rituals.
• The Manipuris believe that martial arts played a crucial role in driving the Burmese out
of their state.
• Thang-ta exhibits close ties to Banshay, a weapon-based martial art from Myanmar.
• Within the art of Thang-ta, three distinct styles were established by Paona Naol Singh,
Ningthoukhongja Poila, and Loukrakpam Sana Mityeng.

Q.58) Solution (a)

Explanation:

Chhau Dance

Chhau dance, originating from the Kalinga region of Odisha and spreading to West Bengal and
Jharkhand, is a unique semi-classical Indian dance that blends martial and folk traditions. In
recognition of its cultural significance, Chhau dance was added to UNESCO's Representative
List of Intangible Cultural Heritage of Humanity in 2010. (Hence statement 3 is correct)

• There are three distinct sub-genres of Chhau based on their place of origin: Purulia
Chhau (West Bengal), Seraikella Chhau (Jharkhand), and Mayurbhanj Chhau (Odisha).
• This dance form is typically observed during the spring festival, spanning a duration of
13 days, with active community participation.
• Male dancers perform the Chhau dance at night in open spaces, combining elements
of dance and martial arts with mock combat techniques.(Hence statement 2 is
incorrect)
• The Chhau dance draws its themes from Hindu mythology and is accompanied by
masks worn by the dancers, except in Mayurbhanj Chhau.
• Chhau dance is an eastern Indian style that reenacts scenes from epics like the
Mahabharata and Ramayana, as well as local folklore and abstract concepts. (Hence
statement 1 is correct)
• This dance form is closely associated with regional festivities; particularly Chaitra Parva
celebrated in the spring.
• Its roots can be traced back to indigenous dance and martial arts traditions.
• The movement language of Chhau dance incorporates mock combat techniques,
stylized gaits inspired by birds and animals, and gestures influenced by the daily
activities of village housewives.
• Performed by male dancers in open spaces during the nighttime, Chhau dance is
accompanied by traditional and folk music played on instruments such as the mohuri
and shehnai reed pipes.
.
IASBABA’S PRELIMS TEST SERIES (AIPTS) 2024 – TEST 7 (ART & CULTURE) SOLUTIONS

• The music ensemble prominently features the resonant beats of multiple drums.

Q.59) Solution (b)

Explanation:

UNESCO Creative Cities Network (UCCN)

The UNESCO Creative Cities Network (UCCN) was established by UNESCO in 2004 to encourage
collaboration among cities that recognize creativity as a crucial element for sustainable urban
development. The Network encompasses seven creative fields: Crafts and Folk Arts, Media
Arts, Film, Design, Gastronomy, Literature, and Music. As of November 2021, six Indian cities
have been included in the UNESCO Creative Cities Network (UCCN):

• Srinagar - Crafts and Folk Arts (2021)


• Hyderabad - Gastronomy (2019)
• Mumbai - Film (2019)
• Chennai - Music (2017)
• Jaipur - Crafts and Folk Arts (2015)
• Varanasi - Music (2015)

Q.60) Solution (a)

Explanation:

Mohiniyattam

• Mohiniyattam, or the Dance of an Enchantress ('Mohini' means beautiful woman and


'attam' means dance), is a solo dance performance by women that was further refined
by Vadivelu in the 19th century and rose to prominence under the monarchs of
Travancore in what is now Kerala.
• Swathi Thirunal, the Travancore ruler in the nineteenth century, was a renowned
patron.
• After it had faded into obscurity, it was resurrected by the famed Malayali poet V. N.
Menon and Kalyani Amma.
• At the 11th century Vishnu temple at Thrikodithanam, Kottayam, there is statues of
female dancers in positions similar to those seen in Mohiniyattam.
• Mohiniyattam is mentioned in the Vyavahara Mala written by Mazha mangalam
Narayanan Namboodi.

Features of Mohiniyattam
.
IASBABA’S PRELIMS TEST SERIES (AIPTS) 2024 – TEST 7 (ART & CULTURE) SOLUTIONS

• Mohiniyattam, a classical dance form, combines the grace of Bharatnatyam and the
vigor of Kathakali, featuring a soft footwork without heavy foot stomping.
• The dance form generally depicts Vishnu's feminine dance and encompasses Nritta
and Nritya elements. (Hence statement 1 is correct)
• Lasya, representing beauty and elegance, takes precedence in Mohiniyattam, making
it predominantly performed by female dancers.
• The costume in Mohiniyattam holds significance, with white and off-white colors and
gold brocade motifs being prominent.
• Minimal makeup is applied to the face, while the dancer wears a leather strap with
bells (Ghungroo) on the ankles.
• Acting plays a crucial role, allowing the dancer to connect with characters and evoke
various emotions through compositions like Padams and Pada Varnams.
• The hand movements, totaling 24, draw inspiration from Kathakali's Hastha Lakshana
Deepika, as well as Natya Shastra, Abhinaya Darpana, and Balarama Bharatham.
• Facial expressions and gestures in Mohiniyattam are more natural and realistic,
reflecting a gramya and lokadharmi style.
• The dance form symbolizes the element of air and includes a collection of forty basic
movements known as 'Atavakul or Atavus.'
• Mohiniyattam is accompanied by Carnatic music, with lyrics in Manipravalam, a
language blending Sanskrit and Tamil/Malayalam. (Hence statement 2 is incorrect)
• Flute, veena, and percussion instruments such as Mridangam, Maddalam, Idakka,
and Kuzhitalam are utilized in its musical accompaniment. (Hence statement 3 is
correct)
• Most of the songs performed in Mohiniyattam follow the Sopana style.

Q.61) Solution (b)

Explanation:

Qawwali

• Qawwali is a devotional music form that praises Allah, the Prophet Muhammad, and
Sufi saints in Islam.
• It typically features a single raga and is predominantly written in Urdu, Punjabi, or
Hindi, with some words from Brajbhasa and Awadhi. (Hence statement 1 is incorrect)
• Qawwali performances take place in Sufi shrines, with solo or group singers
accompanied by a team of around eight members. (Hence statement 2 is correct)
• Musical instruments such as tabla, dholak, and harmonium are used, and the tempo
gradually builds up to create a transcendent experience.
.
IASBABA’S PRELIMS TEST SERIES (AIPTS) 2024 – TEST 7 (ART & CULTURE) SOLUTIONS

• While Amir Khusrau is credited as the originator of Qawwali, this claim has been widely
debated. Notable Qawwals include the Sabri Brothers, Nusrat Fateh Ali Khan, and Aziz
Warsi.
• Amir Khusrau is also known for inventing the Sehtar, a three-stringed musical
instrument.

Q.62) Solution (a)

Explanation:

Hindustani music and Carnatic music are two distinct classical music traditions in India. Here
are some key differences between them:

Origin and Historical Development:

• Hindustani music originated in North India, while Carnatic music originated in South
India.
• Hindustani music has been influenced by Persian and Islamic music traditions, while
Carnatic music has a strong foundation in ancient Indian musical systems. (Hence
statement 1 is correct)

Raga System:

• Both traditions use a system of ragas (melodic frameworks), but the raga structures
and scales differ.
• Hindustani music has a greater emphasis on improvisation, allowing for more flexibility
and exploration within the ragas.
• Carnatic music follows a more structured approach with well-defined compositions
and precise rules for improvisation. (Hence statement 2 is incorrect)

Melodic Structure:

• Hindustani music is characterized by the use of meend (gliding between notes),


gamakas (ornamentation), and improvisation in the alap section.
• Carnatic music focuses on intricate melodic patterns, intricate gamakas (known as
sangatis), and rhythmic variations.

Talas (Rhythm):

• Hindustani music follows a diverse range of talas (rhythmic cycles), including both
simple and complex rhythmic patterns.
• Carnatic music employs a unique system of talas called "adi talas" and "jathi talas,"
which have specific rhythmic structures and mathematical calculations.

Instruments:
.
IASBABA’S PRELIMS TEST SERIES (AIPTS) 2024 – TEST 7 (ART & CULTURE) SOLUTIONS

• Hindustani music commonly uses instruments such as sitar, sarod, tabla, harmonium,
and flute.
• Carnatic music is often accompanied by instruments like veena, violin, mridangam,
ghatam, and kanjira. (Hence statement 3 is correct)

Performance Style:

• Hindustani music includes various forms like dhrupad, khayal, thumri, and ghazal, and
the performances are often marked by improvisation and emotive renditions.
• Carnatic music includes compositions in different musical forms like varnam, kriti,
padam, and javali, and the focus is on intricate melodic patterns and rhythmic
precision.

Q.63) Solution (b)

Explanation:

Ramappa temple

• Ramappa temple is situated in


Palampet in Telangana. The 13th-
century Ramappa temple has been
recently inscribed as a UNESCO World
Heritage Site in 2021. In a virtual
meeting of the heritage committee, 17
countries backed the move to ensure the
heritage stack for the site. Ramappa
Temple was nominated as UNESCO
World Heritage Site in 2014. Warangal
on the Deccan Plateau is popular for its
classic and splendid Kakatiya art.
• The Ramalingeswara Temple, which is situated in Warangal, commonly known as the
Ramappa Temple, is one such wonderful work of art that stands as a testament to the
Kakatiya dynasty of, the royal family. The temple was named Ramappa after its main
sculptor, probably the only temple in the country known by its sculptor’s name.
• After being released from the rule of the Chalukyan dynasty, the Kakatiya dynasty ruled
most of central India to the east coast in the 12th and 14th centuries. The construction
of a Ramappa temple started in 1213 A.D. during the reign of Recharla Rudra, who was
the general of Ganpati Deva, a Kakatiya king.
• Rudreshwar or Ramappa temple is a sandstone temple dedicated to Ramalingeswara
Swamy (Shiva). (Hence statement 2 is correct)
.
IASBABA’S PRELIMS TEST SERIES (AIPTS) 2024 – TEST 7 (ART & CULTURE) SOLUTIONS

• It is the main Shiva temple in a larger walled temple complex, which includes several
smaller temples and structures. Sculptor Ramappa built the Ramappa temple over 40
years.
• Ramappa temple is close to the shores of a Kakatiya-built water reservoir- Ramappa
Cheruvu.
• The Ramappa Temple showcases exquisite craftsmanship, using rare black basalt and
sandstone materials. It features lightweight brick construction and took around 40
years to complete.
• The temple follows the stellate architectural form and the Vesara style of vimana
popularized by the Chalukyas. It stands on a 6-foot-high platform with a cruciform
plan. The temple room is covered by a Shikharam and is surrounded by intricate
carvings. (Hence statement 1 is incorrect)
• At the entrance, there is a Nandi Mandapam with a majestic Nandi Vigraham statue.
The Kakatiyas demonstrated their expertise in construction and geo-technology by
using sand in the foundations to create earthquake-resistant structures. (Hence
statement 3 is correct)
• Despite a lack of perennial water sources, the Kakatiyas implemented well-planned
irrigation systems, a characteristic feature of their kingdom. The Telangana
government initiated "Mission Kakatiya" to restore tanks and irrigation networks,
drawing inspiration from the Kakatiya dynasty.
• During the 14th century, Telugu literature flourished under the Kakatiya rule. The
dancing female figures carved in the Ramappa Temple inspired Jayasenapati to write
Nritta Ratnavali, a significant literary work.
• The dancing sculptures in the Rudreshwara Temple provide insights into traditional
desi dances performed within the temple, such as Perini, Prekhana, Gavundali, Rasaka,
Dandarasaka, and Ghatisisri Nrttam.

Q.64) Solution (d)

Explanation:

Warli paintings

• Warli paintings derive their name from the indigenous Warli people who have
preserved this artistic tradition since ancient times, dating back to 2500-3000 BC.
• Primarily found along the Gujarat-Maharashtra border, the Warlis create intricate
ritualistic paintings depicting scenes of fishing, hunting, farming, dances, animals,
trees, and festivals, with a central focus on a chaukat or chauk. (Hence statement 1 is
correct)
• These paintings feature goddesses like Palaghata and male gods representing spirits in
human form.
.
IASBABA’S PRELIMS TEST SERIES (AIPTS) 2024 – TEST 7 (ART & CULTURE) SOLUTIONS

• Warli art is characterized by its portrayal of flora, wildlife, and festivals, showcasing a
deep connection with nature. Its rhythmic style evokes a sense of timeless beauty
reminiscent of prehistoric paintings.

Features of Warli Painting:

• Depiction of Daily Occurrences: These paintings depict the daily occurrences


experienced by the Warli tribe.
• Focus on Human Figures: Warli art primarily focuses on humans dancing in spirals and
open-ended circles.
• Variety of Mediums: Initially done on walls, Warli art expanded to other mediums such
as bamboo, cloth, mud pots, and dried bottle gourds.
• Color Palette: Originally limited to earth brown and rice paste, Warli art now
incorporates a wider range of colors like Henna, Indigo, Ochre, Black, Earthy mud, and
Brick red.
• Gender Inclusivity: Initially painted by Warli women (Savasini), men also started
practicing Warli painting over time.
• Non-Legendary Depictions: Warli paintings do not depict legendary stories but focus
on various aspects of daily life. (Hence statement 2 is correct)
• Four Categories: Warli paintings can be categorized into Gods, People, Creatures, and
Rights/Rituals, depicting historical folklore, deeds of the people, animals, and daily
activities.
• Chawk Painting: The Chawk painting is a famous depiction where married women
paint white on kitchen walls.
• Goddess Palaghat: The central figure in rectangular space is Goddess Palaghat,
surrounded by trees, men engaged in daily chores, dancers, women's tools, and
animals.
• Geometric Shapes: Warli wall paintings utilize basic geometric shapes such as
triangles, circles, squares, dots, and dashes. (Hence statement 3 is correct)

Q.65) Solution (d)

Explanation:

Rummindei Pillar Inscription:

• Ashoka erected the Rummindei pillar in present-day Lumbini, Nepal, to signify the
birthplace of the Buddha.
• The inscription on the pillar declares Ashoka's visit and his deep devotion to
Buddhism. (Hence 1 is correct)

Bhabhru Edict:
.
IASBABA’S PRELIMS TEST SERIES (AIPTS) 2024 – TEST 7 (ART & CULTURE) SOLUTIONS

• The Bhabhru Edict, also known as the Calcutta-Bairat Edict, focuses on Ashoka's
conversion to Buddhism.
• The edict documents Ashoka's embrace of the Buddhist faith and his support for
theBuddhist Sangha.(Hence 2 is correct)

Schism Edicts:

• The Schism Edicts, discovered at Sanchi and Sarnath, tackle the issue of divisions
within the Buddhist Sangha.
• Ashoka expresses his concern for the unity of the Buddhist community and warns
against creating rifts and schisms. (Hence 3 is correct)

Nigalisagar Pillar Inscription:

• The Nigalisagar pillar inscription, situated in present-day Nepal, commemorates


Ashoka's visit to the site and his expansion of a stupa dedicated to the Buddha.
• The inscription signifies Ashoka's patronage of Buddhism and his efforts to promote
the faith. (Hence 4 is correct)

Q.66) Solution (b)

Explanation:

India is home to several magnificent sun temples dedicated to the worship of the Sun God.
These temples reflect the deep reverence for the Sun that has been present since ancient
times. Some notable sun temples in India include:

• Modhera Sun Temple, Gujarat: Constructed in the 11th century, this temple showcases
intricate architectural beauty.
• Konark Sun Temple, Odisha: Built in the 13th century, it is shaped like a chariot and
features a mandapa on a raised platform.
• Brahmanya Dev Temple, Unao (Madhya Pradesh).(Hence pair 1 is correctly matched)
• Suryanar Kovil, Kumbakonam (Tamil Nadu): Constructed in the 11th century, it follows
the Dravidian architectural style and houses shrines of the Sun God and other celestial
bodies.
• Suryanarayana Swamy Temple, Arasavalli (Andhra Pradesh): Believed to be built in the
7th century, it features a granite idol of the Sun God holding a lotus.
• Dakshinaarka Temple, Gaya (Bihar): Built in the 13th century by King Prataparudra of
Warangal, it has a unique blend of Persian attire in the idol's depiction. (Hence pair 2
is correctly matched)
• Navalakha Temple, Ghumli (Gujarat): Constructed in the 11th century, it exhibits the
Solanki and Maru-Gurjara architectural styles.
• Surya Pahar Temple, Goalpara (Assam).
.
IASBABA’S PRELIMS TEST SERIES (AIPTS) 2024 – TEST 7 (ART & CULTURE) SOLUTIONS

• Martand Sun Temple, Kashmir. (Hence pair 3 is incorrectly matched)


• These sun temples reflect the rich cultural and religious heritage of India, showcasing
the significance of the Sun in Hindu mythology and worship.

Q.67) Solution (b)

Explanation:

Pala and Sena School of Architecture

In the Bengal region, a distinctive style of architecture emerged known as the Pala and Sena
School of architecture. This architectural tradition flourished between the 8th and 12th
centuries AD, under the patronage of the Pala dynasty and Sena dynasty. The Palas, who were
predominantly Buddhist rulers following the Mahayana tradition, exhibited great tolerance
and supported both Buddhism and Hinduism. They constructed numerous Viharas, Chaityas,
and Stupas. The Senas, on the other hand, were Hindu rulers who built temples dedicated
to Hindu deities while also fostering Buddhist architecture. As a result, the architecture of the
region displayed a blend of both religious influences. (Hence statement 1 is correct)

Monuments associated with the Pala rulers include:

• Nalanda, Jagaddala, Odantapuri, and Vikramshila Universities. (Hence statement 3 is


incorrect)
• The Somapura Mahavihara, an impressive monastery located in present-day
Bangladesh.

Monuments associated with the Sena rulers include:

• The Dhakeshwari Temple in Bangladesh. (Hence statement 4 is incorrect)

The architectural style of the Sena

• The architectural style of the Sena rulers was characterized by curved or sloping roofs
resembling bamboo huts, which later became known as the "Bangla roof" and was
adopted by Mughal architects. (Hence statement 2 is correct)
• The principal building materials used were burnt bricks and clay, particularly terracotta
bricks.
• The temples in this region featured tall, curving shikharas (spires) crowned by large
amalakas, reminiscent of the Odisha School of architecture.
• Both stone and metal were used for sculptures in this region, with stone being the
predominant material. The sculptures were notable for their highly polished finish.
• Examples of these architectural and sculptural marvels include the Siddhesvara
Mahadeva Temple in Barakar and the temples surrounding Bishnupur in West Bengal.
.
IASBABA’S PRELIMS TEST SERIES (AIPTS) 2024 – TEST 7 (ART & CULTURE) SOLUTIONS

Q.68) Solution (d)

Explanation:

Dashavatara Vishnu Temple, Deogarh, Uttar Pradesh

• The temple's patrons and benefactors are unknown, but it is believed to have been
constructed in the early sixth century CE. (Hence statement 1 is correct)
• It is an excellent example of late Gupta architecture.
• The temple follows the Panchayatana style of architecture, featuring a main shrine on
a rectangular plinth and four smaller subsidiary shrines at the corners, totaling five
shrines. (Hence statement 2 is correct)
• The temple walls showcase three main reliefs of Vishnu, while the original dedications
of the four secondary shrines remain unclear. (Hence statement 3 is correct)
• The statues of Ganga and Yamuna flank the grand west-facing gateway of the temple.
• The shikhara (spire) follows the Latina/Prasada style, indicating its early association
with the nagara style of temple architecture.
• Inside the temple, there are depictions of Sheshayana (Vishnu reclining on
Sheshanaga), Nara-Narayana (discussion between the human soul and the eternal
divine), and Gajendra Moksha (the story of Vishnu subduing an asura in the form of an
elephant).
• The temple's west-facing orientation is uncommon, as most temples typically face east
or north.

Q.69) Solution (c)

Explanation:

• There is no national language of India. Hindi is not a national language. Neither the
Constitution nor any Act defines the word 'national language.
• The Constitution does not specify an official language for the States for conduct of
their official functions. States are free to adopt an official language. (Hence
statements 1 and 2 are correct)
• The language to be adopted by the States need not be one of those listed in the Eighth
Schedule. Several States have adopted an official language which is not listed in the
Eighth Schedule. (Hence statement 3 is incorrect)
• English is not in the list of 22 scheduled languages as per the Eighth Schedule of the
Indian Constitution.
• Arunachal Pradesh and Nagaland are the only states which have English as their only
official language.
.
IASBABA’S PRELIMS TEST SERIES (AIPTS) 2024 – TEST 7 (ART & CULTURE) SOLUTIONS

Q.70) Solution (a)

Explanation:

Some of the features of shadow puppetry are as follows:

India has a rich tradition in shadow puppetry


that has persisted over time.

• Shadow puppets used in this art form are


flat figures made from cut-out leather. (Hence
statement 1 is correct)
• The figures are painted identically on
both sides of the leather. (Hence statement 2
is incorrect)
• During performances, the puppets are
placed on a white screen, with light illuminating them from behind, creating shadows
on the screen.
• Skilled puppeteers manipulate the figures to bring their silhouettes to life, telling
captivating stories.
• Examples of Indian shadow puppetry include Tolpavakuthu, Togalu gombeyatta, and
Tolu Bommalata. (Hence statement 3 is correct)

Q.71) Solution (b)

Explanation:

Classification of Indian Languages

Languages in India are classified into the following major sub-groups:

Indo-Aryan Group of Languages:

• It is a branch of the larger Indo- European family which came to India with the advent
of the Aryans. It is the largest language group of India and around 74% of the Indians
speak those languages which belong to this group. This language group is further
divided into 3 sub-groups depending upon the time period of their origin. (Hence
statement 1 is correct)
o Old Indo-Aryan Group – Sanskrit.
o Middle Indo-Aryan Group - Prakrit, Ardha-Magadhi, Pali and Apabhramsa.
o Modern Indo-Aryan Group - Hindi, Assamese, Bengali, Gujarati, Marathi, Punjabi,
Rajasthani, Sindhi, Odia, Urdu, etc. belong to this group.

The Assamese belongs to Indo-Aryan group of languages. (Hence statement 2 is incorrect)


.
IASBABA’S PRELIMS TEST SERIES (AIPTS) 2024 – TEST 7 (ART & CULTURE) SOLUTIONS

Dravidian Group of Languages:

• This group mainly comprises thelanguages spoken in the southern part of India.
Around 25% of the Indian population is covered under this group. Proto Dravidian gave
rise to 21 Dravidian languages. They can be broadly classified into 3 groups:
o Northern Group – Brahui, Malto and Kurukh.
o Central Group – Gondi, Khond, Kui, Manda, Parji, Gadaba, Kolami, Pengo, Naiki,
Kuvi and Telugu.
o Southern Group – Kannada, Tamil, Malayalam, Tulu, Kodagu, Toda and Kota

Sino-Tibetan Group of Languages:

• Languages under this group belong to Mongoloid family and stretch to all over the
Himalayas, North Bihar, and North Bengal, Assam and up to the North- Eastern
frontiers of the nation. These languages are considered to be older than the Indo-
Aryan Languagesand are referred to as Kiratas to in the oldest Sanskrit literature.
Languages belonging to this group are spoen by 0.6 percent of the Indian population.
This group is further divided into the following:
o Tibeto-Burman - Sikkimese, Bhutia, Balti, Sherpa, Lahuli, Kinnauri, Limbu, Abor.
Miri, Aka, Dafla, Mishmi, Garo, Bodo, Mikir, Naga, Meitei, etc.
o Siamese-Chinese – Ahom is one ofthe languagess which belong to this group.
However, this language is already extinct in the Indian sub-continent.

Austric Group of Languages:

• Languages under this group belong to the Austro-Asiatic sub-family which are
represented by the languages of Munda or Kol group and spoken in Central, Eastern,
and North-Eastern India. The existence of these languages has been much before the
advent of the Aryans and was referred to in ancient Sanskrit literature as Nisadas.
(Hence statement 3 is correct)
• Santhali is the most important language in this group. With the exceptions of Khasi
and Santhali, all Austro-Asiatic languages on Indian Territory are endangered.

Q.72) Solution (c)

Explanation:

Vaishnavism: The followers consider Vishnu as the Supreme Lord. The tradition has traceable
roots to the 1st millennium BC, as Bhagavastism, also called Krishnaism. The Vaishnava
tradition has many sampradayas or sub-schools.

Prominent Sects under Vaishnavism


.
IASBABA’S PRELIMS TEST SERIES (AIPTS) 2024 – TEST 7 (ART & CULTURE) SOLUTIONS

• Varkari Panth or Varkari Sampradaya: The followers in this community are devotees
of Lord Vishnu, in his manifestation as Vithoba, and the worship is centred in Vithoba
Temple at Pandharpur in Maharashtra. The sect strictly abstains from alcohol and
tobacco. The annual pilgrimage, Vari, involves interesting events. In the Vari, the
Varkaris carry the padukas of the saints in palkis from Samadhi to Pandharpur. The
events Ringan and Dhava are held during the pilgrimage. During the Ringan, a sacred
horse runs through the rows of pilgrims, who try catching the dust particles kicked off
and smear their heads with the same. Prominent figures under this sect include
Jnaneshwar, Namdev, Eknath and Tukaram. (Hence statement 1 is correct)
• Ramanandi Sampradaya: The followers adhere to the teachings of the Advaita scholar
Ramananda. This is the largest monastic group within Hinduism, and these Vaishnava
monks are known as Ramanandis, Vairagis or Bairagis. They worship Rama, one of the
ten incarnations of Vishnu. These ascetics perform meditation and follow strict ascetic
practices, but they also believe that the grace of god is required to achieve liberation.
They are mainly settled around the Gangetic plains. Its two sub-groups are Tyagi and
Naga. (Hence statement 2 is incorrect)
• Brahma Sampradaya: It is also known as the MadhvaSampradaya or the Brahma-
Madhva-Gaudiya Sampradaya, is one of the four main Vaishnava sampradayas
(spiritual lineages). It traces its lineage back to the teachings of Madhvacharya (also
known as Acharya Madhva), who is considered the founder of the sampradaya. The
Brahma Sampradaya emphasizes the doctrine of Dvaita (dualism), which asserts the
eternal distinction between the individual souls (jivas) and the Supreme Lord Vishnu.
The followers of the Brahma Sampradaya worship Lord Vishnu as the supreme deity,
and their practices include devotional chanting, study of Vedic scriptures, and the
performance of rituals and ceremonies. The sampradaya has had a significant
influence on the religious and cultural traditions of the Karnataka region in India,
where Madhvacharya lived and taught.ISKCON belongs to this sampradaya. (Hence
statement 3 is correct)
• Pushtimarg Sampradaya: It is a Vaishnav sect founded by Vallabhacharya around 1500
AD. Its philosophy is that the ultimate truth is one and only one Brahm. The devotion
is based on pure love for Lord Krishna. All followers are expected to perform seva to
their personal idol of Krishna.
• Nimbarka Sampradaya: It is also known as the Hamsa Sampradaya, Kumara
Sampradaya or Sanakadi Sampradaya, the followers worship Radha and Krishna
deities

Q.73) Solution (b)

Explanation:

Zoroastrianism
.
IASBABA’S PRELIMS TEST SERIES (AIPTS) 2024 – TEST 7 (ART & CULTURE) SOLUTIONS

• This religion has its origin in Persia and was founded by the prophet Zarathustra
around the 7th centuryBC. It is a monotheistic religion which believes in one eternal
god named Ahura Mazda, who is the epitome of just behaviour and goodness. (Hence
statement 1 is correct)
o There is an opposing spirit of evil and bad behaviour called Angra Mainyu.
These two forces keep fighting with each other in an eternal struggle, and one
day good will win over evil, and that will be the last day. (Hence statement 2 is
incorrect)
o The first contact of the Zoroastrians with India was probably between the 8th
and 10th century AD, when they fled from Iran because of the Islamic invasions.
They are commonly known as Parsis and Iranis and are presently amongst the
smallest (and most rapidly shrinking) communities in India. They live mostly in
Mumbai, Goa and Ahinedabad.
• In India, the Iranis are a smaller Zoroastrian community than the Parsis and are recent
arrivals compared to the Parsis.
• Their sacred text called the Zend Avesta is written in Old Avestan and consists of 17
sacred songs (gathas) and the Ahuna Vairyo (sacred chant), which was supposedly
written by Zarathustra himself. The translations of these texts and compiled glossaries
are called Zend. (Hence statement 3 is correct)

This collection is divided into five parts:

• Yasna: describes worship through ceremony and offerings.


• Vendidad: describes laws against the demons or evil spirits.
• Yashts: contains hymns of worship through praise.
• Khordeh Avesta: book of common prayers.
• Gathas: These are a part of the Yasna texts and are further divided into five parts called
the Ahunavaiti, Ushtavaiti, Spenta-mainyu, Vohte-khshathra and Vahishtaishti.

They worship fire and consider fire and earth to be sacred elements. They believe that dead
matter is a corrupting element to everything, and hence, they place their dead bodies in the
open to be eaten by vultures. These open spaces are called 'dakhma' or tower of silence, and
the vultures that eat them are called 'Dakhma-Nashini'. The only space in India where they
are left are the ‘Tower of Silence' in Mumbai. Nowadays, people have even started burying
their dead due to Indian vulture crisis which has led to slower decomposition of bodies.

Their fire temples called Atash Behram are rare and only eight known temples exist in the
entire country.

There are three major calendar forms used by the Parsis:

• Shahenshahi (majority): Uses the coronation of the last Sasanian ling. Yazdegerd III as
the first year for dating the calendar.
• Qadimi: Claimed to be the oldest calendar of the three.
.
IASBABA’S PRELIMS TEST SERIES (AIPTS) 2024 – TEST 7 (ART & CULTURE) SOLUTIONS

• Fasli: Claimed to be the most accurate religious calendar and the most recent of the
three.

Q.74) Solution (b)

Explanation:

Shramana Schools

The word Shramana means one who performs act of austerity and ascetic. It refers to several
Indian religious movements parallel to Vedic religion.

The various Shramana Schools include the following:

1. Jainism
2. Buddhism
3. Ajivika (Hence statement 1 is correct)
4. Ajnana
5. Charvaka

All five schools mentioned above belong to the Nastika or heterodox school of philosophy.

Ajivikas

• The school was founded by Makkhali Gosala in the 5th century BC.
• The school revolves around the Niyati (fate) doctrine of absolute determinism. It
believes that there is no free will and that whatever has happened is happening or will
happen is entirely pre-ordained or pre-decided and based on cosmic principles. Hence
there was no use for karma.
• It is based on the theory of atoms and believes that everything is composed of atoms
and the various qualities emerge from the aggregates of atoms which is pre-
determined. The Ajivikas led a simple ascetic life, without clothes and any material
possession.
• They opposed Buddhism and Jainism and were atheists.
• They did not believe in the doctrine of karma unlike the followers of Jainism and
Buddhism. They consider karma to be a fallacy. (Hence statement 2 is incorrect)
• They also rejected the authority of the Vedas, just like the followers of Buddhism and
Jainism.
• However, the Ajivikas believed in the existence of a soul (atman) in every living being,
just like the Jains. They believed in the existence of a soul in material form, whereas
Jainism propounds a formless soul.
• Bindusara (4th century BC) was one of its followers.
• Savatthi (Shravasti) in Uttar Pradesh is believed to be the centre of Ajivika.(Hence
statement 3 is correct)
.
IASBABA’S PRELIMS TEST SERIES (AIPTS) 2024 – TEST 7 (ART & CULTURE) SOLUTIONS

• The edicts of Ashoka's seventh pillar mention the Ajivikas.


• The texts of the Ajivika sect are non-existent at present. This sect has also lost its
glamour in the present era.

Q.75) Solution (c)

Explanation:

Bodhisattva in Buddhism

• Bodhisattva is the one who has generated bodhicitta (a spontaneous wish and a
compassionate mind) to attain Buddhahood for the benefit of all sentient beings. It
believes in universalliberation and is a concept under Mahayana Buddhism. (Hence
statement 1 is incorrect)
• Buddha in his previous births was thought to be a bodhisattva as contemplated in
the Jataka tales, and the stories depict the various attempts of the bodhisattva to
embrace qualities like self-sacrifice and morality.
• Under Theravada Buddhism, a person whose aim is to become fully enlightened is still
subject to birth, illness, death, sorrow, defilement and delusion.
• On the path to become a Buddha, a bodhisattva proceeds through 10 grounds or
bhumis, namely great joy, stainless, luminous, radiant, very difficult to train, obviously
transcendent. Gone afar, immovable, good discriminating wisdom and cloud of
Dharma.
• By passing these 10 bhumis, he becomes an enlightened one.

Prominent Bodhisattvas under Buddhism include the following:

Avalokitesvara, Vajrapani and Manjusri are the three protective deities around Buddha.
(Hence statement 2 is correct)

• Avalokitesvara: One of the three protective deities around Buddha. He is described as


holding a Lotus Flower and is also known as Padmapani. His painting can be found at
the Ajanta Caves and is the most acknowledged among all the Bodhisattvas. The
bodhisattva of compassion, the listener of the world's cries who uses skilful means to
come to their aid. He appears unofficially in Theravada Buddhism in Cambodia under
the name Lokesvara.
• Vajrapani: One of three protective deities around Buddha and is also depicted in the
Ajanta Caves. Vajrapani is considered to manifest all the powers of Buddha as well as
the power of all five tathagatas, namely Vairocana, Akshobhya, Amitabha,
Ratnasambhava and Amoghasiddhi.
• Manjusri: One of the three protective deities around Buddha and is also depicted in
the Ajanta Caves. He is associated with the Wisdom of Buddha and is a male
Bodhisattva with a wielding sword in his hand.(Hence statement 3 is correct)
.
IASBABA’S PRELIMS TEST SERIES (AIPTS) 2024 – TEST 7 (ART & CULTURE) SOLUTIONS

• Samantabhadra: Is associated with practice and meditation. Together with the Buddha
and Manjusri, he forms the Shakyamuni trinity in Buddhism.
• Ksitigarbha: He is depicted as a Buddhist monk and took a vow not to achieve
Buddhahood till hell is completely emptied.
• Maitreya: A Future Buddha who will appear on Earth in the future, achieve complete
enlightenment and teach pure dharma. Laughing Buddha is said to be an incarnation
of Maitreya.
• Akasagarbha: Is associated with the element of space.
• Tara: Is associated with Vajrayana Buddhism and represents the virtues of success in
work and achievements.
• Vasudhara: Is associated with wealth, prosperity and abundance. Popular in Nepal.
• Skanda: Guardian of the viharas and the Buddhist teachings.
• Sitatapatra: She is considered a protector against supernatural danger and is
worshipped in both Mahayana and Vajrayana traditions.

Dipankar Buddha: Dipankara was one of the Buddhas of the past who reached enlightenment
prior to Gautama Buddha. As per Buddhism, Dipankara was a previous Buddha, Gautama
being the most recent Buddha and Maitreya to be the future Buddha. He is respected under
all three sub-sects of Buddhism - Mahayana, Vajrayana and even Theravada.

Q.76) Solution (d)

Explanation:

Vajrayana Buddhism:

• It is also known as Tantric Buddhism.


• Scholars contemplate that Vajrayana School developed as a result of royal courts
sponsoring both Buddhism and Shaivism, i.e., they say it was influenced by Hinduism.
• The main deity is Tara (a lady).
• It involved merging Brahmanical (Veda-based) rites with Buddhist philosophies.
(Hence statement 1 is correct)
• Vajrayana is based on Mahayana Buddhist philosophy. (Hence statement 2 is correct)
• This school believes in Tantra, Mantra and Yantra superiority as being a faster vehicle
to liberation containing many skilful methods of tantric ritual.
• It includes practices that make use of mantras, dharanis, mudras, mandalas and the
visualization of deities and Buddhas. It also combines Mahayana ideals with a body
of esoteric knowledge and tantric practice.
• According to this school, the mantra is an easy path to achieve Buddhahood without
the difficulties as compared to striving for six perfections or Paramitas under
Mahayana.
.
IASBABA’S PRELIMS TEST SERIES (AIPTS) 2024 – TEST 7 (ART & CULTURE) SOLUTIONS

• 5.7 per cent of the world's Buddhist population follows it.


• Countries following it include Tibet, Bhutan, Mongolia etc. (Hence statement 3 is
correct)

Q.77) Solution (d)

Explanation:

The Tipitakas

The Tipitaka (in Pali) and Tripitaka (in Sanskrit) meaning ‘The Three Baskets/Collections' is
one of the earliest compilations of Buddhist teachings written (originally) on long, narrow
leaves. The Buddhist monks transcribed and categorised the Buddha's teachings in three
baskets, which is how the name "Three Baskets' originated. All branches of Buddhism have
the Tripitaka as part of their core scriptures, which comprises three books.

• The Sutta Pitaka (Sutra/Basket of Discourses): This contains the Buddha's discourses
on various doctrinal issues in dialogue form. These texts are also known as 'Buddha
Vacana' or 'the word of the Buddha', as it refers to texts that are supposed to contain
what the Buddha himself said. With the exception of few sutras, the authority of this
text is accepted by all Buddhist schools. (Hence statement 1 is correct)
• These discourses were arranged on the basis of the manner in which they were
delivered.
• The suttas are divided into five categories, or nikayas.
o Digha Nikaya
o Majjhima Nikaya
o Samyutta Nikaya
o Anguttara Nikaya
o Khuddaka Nikaya
• The Vinaya Pitaka (Discipline Basket): This contains rules for monks and nuns of the
monastic order (Sangha). It includes the Patimokka - a list of transgressions against
monastic discipline and atonements for these. Apart from monastic rules, the Vinaya
texts also include doctrinal expositions, ritual texts, biographical stories and some
elements of the 'Jatakas' or 'birth stories'. (Hence statement 2 is correct)
• This pitaka is divided into three sections.
o Suttavibhanga
o Khandhaka
o Parivara
• The Abhidhamma Pitaka (Basket of Higher Teachings): This contains a thorough study
and systemisation of the teachings of the Sutta Pitaka through summaries, questions
and answers, lists, etc. (Hence statement 3 is correct)
• There are seven books in the Abhidhamma Pitaka.
.
IASBABA’S PRELIMS TEST SERIES (AIPTS) 2024 – TEST 7 (ART & CULTURE) SOLUTIONS

o Dhammasangani
o Vibhanga
o Dhatukatha
o Puggalapannatti
o Kathavatthu
o Yamaka
o Patthana

Q.78) Solution (c)

Explanation:

Saka Samvat: This calendar form is believed to have been initiated by King Shalivahana in 78
AD. It was also known as the Saka era as it is to this tribe that Shalivahana belonged.

• Like Vikram Samvat, the Saka Calendar is also both solar and lunar, with lunar months
and a solar year, and has the same number of months as the Vikram era has.(Hence
statement 1 is correct)
• However, the months commence in different periods here. Its zero year begins near
the vernal equinox of the year 78 AD. The Saka Calendar begins on 22nd March every
year except in Gregorian leap years when it starts on 21st March.(Hence statement 2
is correct)
• Saka calendar has a year with a fixed number of days in each month. However, the
names of months in both calendars are the same. The Saka calendar starts with Chaitra
followed by Vaisakha, Jyaistha, Asadha, Shravana, Bhadra, Asvina, Kartika, Agrahayana,
Pausa, Magha and Phalguna. The number of days in a Saka year is 365.
• The Saka Calendar used as the official civil calendar in the country is the National
Calendar of India. It is used, through notification in the Official Gazette by the
Government of India, in news broadcasts by All India Radio, and in calendars and
communication documents issued under the control of the Government of India. The
Saka calendar, which is one of the Hindu calendars was originally named as Saka
Samvat and used for the calculation of days of religious significance in the Hindu
Religion.
• The Saka calendar was adopted as the National Calendar in the year 1957 by the
CalendarReforms Committee set up by the Government of India. The Committee
made efforts to coincide the astronomical data and harmonise the usage of this
calendar after rectification of some local errors. (Hence statement 3 is correct)
• It came into use from 22 March 1957, according to the Gregorian calendar which was
actually 1 chaitra 1879 according to the Saka Samvat. It was adopted as the National
Calendar of India in order to synchronize the usage 30 different kinds of calendars in
use in India at the time.
.
IASBABA’S PRELIMS TEST SERIES (AIPTS) 2024 – TEST 7 (ART & CULTURE) SOLUTIONS

Q.79) Solution (a)

Explanation:

Sangeet Natak Akademi

• The Sangeet Natak Akademi (SNA) is the first national academy for the performing Arts
to be set up by the Government of India (1952) and was inaugurated in 1953 by the
first President of India, Dr. Rajendra Prasad. It is the primary body for showcasing the
various performing arts of the country and has the ardent task of promoting the
enormous intangible heritage of India expressed through forms of music, dance and
drama. (Hence statement 1 is correct)
• It is not only the central agency that monitors the preservation of our cultural heritage
but it also collaborates with state and union territory governments to preserve and
promote their culture on a national platform. (Hence statement 2 is incorrect)
• The SNA also establishes and looks after several institutions that focus primarily on
dance, music or drama.

The SNA also collaborates with international organisations such as UNESCO to save the
cultural heritage of India. Its several aims and objectives are as follows:

• To promote research and performances in the fields of music, dance and drama.
• To encourage the establishment of centres for theatres, especially for regional theatres
and to promote teaching acting, study of stagecraft and play production and
direction.
• To promote documentation of literature on Indian music, dance and drama. Special
emphasis should be given to create handbooks and illustrated dictionaries that explain
technical terms.
• To revive, promote and preserve folk dance, music and drama.
• To foster cultural contact with the international community in the fields of dance,
music and drama.

Q.80) Solution (b)

Explanation:

Bathukamma Festival

• Bathukamma is a colourful and vibrant festival of Telangana and celebrated by women,


with flowers that grow exclusively in each region. This festival is a symbol of
Telangana’s cultural identity.
.
IASBABA’S PRELIMS TEST SERIES (AIPTS) 2024 – TEST 7 (ART & CULTURE) SOLUTIONS

• Bathukamma that means ‘Mother Goddess come Alive’ is celebrated towards the end
of monsoon and heralds the abundance that nature has to offer. The festival begins on
the day of Mahalaya Amavasya and is celebrated with much fanfare for nine days. The
final day of Bathukamma, known as Pedha or Saddula Bathukamma falls two days
before Dussehra.
• Bathukamma is a beautiful flower stack, arranged with different unique seasonal
flowers most of them with medicinal values, in seven concentric layers in the shape of
temple gopuram.
• Bathukamma celebrates the inherent relationship between earth, water, and human
beings.

(Hence option (b) is correct answer)

Q.81) Solution (c)

Explanation:

Atharva Veda:

• This Veda is also known as the Brahma Veda and has been attributed to two rishis
called Atharvan and Angiras. Because of its association with the two rishis, in the olden
times, it was also called Atharvangirasa.
• While it is mostly concerned with the peace and prosperity of human society and
covers all aspects of a man's daily life, it specifically focuses on the treatment of
several ailments. The book is known to prescribe treatment for almost 99
diseases.(Hence statement 1 is correct)
• There are two major recessions (sakhas) of the text called the Paippalada and
Saunakiya.
• Most of the text deals with healing and black and white magic, and speculation on the
changes in the universe, and the text even touch upon issues such as the
everydayproblems in a householder's life.

Puranas:

• As the word 'Purana' suggests, these texts talk about 'that which renews the old'.
• The Puranas were written to illustrate and expound the truth of the Vedas.(Hence
statement 2 is correct)
• These are ancient Indian mythological texts, which consist of narrative stories about
the creation of the universe and illustrate its history till the supposed destruction of
the universe.
• It contains the stories of the kings, heroes, sages and demigods, but it focuses on the
divine Hindu Trimurti or trinity/three Gods: Brahma, Vishnu and Mahesh.
.
IASBABA’S PRELIMS TEST SERIES (AIPTS) 2024 – TEST 7 (ART & CULTURE) SOLUTIONS

• There are 18 major Puranas (Mahapuranas), and each gives prominence to a particular
deity and expounds on the philosophical and religious concepts related to that diety.
• Some of the more prominent and well-known Puranas are Bhagavata, Brahma, Vayu,
Agni, Garuda, Padma, Vishnu and Matsya. These contain anecdotes about the social,
cultural and religious life of post-Vedic India and provide historians with critical
information about geography, history and dynastic genealogies.
• These Puranas are written in the form of stories, which combine myths, legends and
sermons about the deities. This easy form of story writing made it very popular
amongst the masses, who did not always understand the complex Vedas.
• The Puranas were translated and distributed in various vernacular languages. They use
parables and fables to spread their message

Upanishads:

• Interestingly, the term Upanishad or u (at), pa (foot), ni (down) and s(h)ad (to sit), i.e.
to sit down near (the teacher), describes the text completely.
• A collection of more than 200 Upanishads are known, and the teacher usually passed
them down verbally to his students in the forest while they sat in front of him. This
tradition was part of the guru-shishya parampara.
• These are treatises written in Sanskrit and give an account of the Vedas in
predominantly monastic and mystical terms. As they are generally the last part of the
Vedas, they are also known as Vedanta or 'end (anta) of the Veda'.
• The Upanishads are said to hold the 'truth' about human life and show the way
towards human salvation or moksha. They continue to talk about the abstract and
philosophical problems faced by mankind, especially the origin of this universe, the
supposed origin of mankind, the life and death cycle, and the material and spiritual
quests of man.
• This is the literature in which ancient sages realised that in the final analysis, man has
to know himself.(Hence statement 3 is correct)
• Out of the above-mentioned 200 Upanishads, a set of 108 Upanishads has been called
the Muktika Canon. This is supposed to be an important canon as the number 108 is
equivalent to the number of beads on a Hindu rosary or mala.
• The teachings propounded in the Upanishads have been a part of the founding rituals
of Hinduism.

Q.82) Solution (a)

Explanation:
.
IASBABA’S PRELIMS TEST SERIES (AIPTS) 2024 – TEST 7 (ART & CULTURE) SOLUTIONS

• Maktabs: Maktabs, also known as maktabas or kuttab, are traditional Islamic


elementary schools where children are taught to read and recite the Quran. These
schools focus on providing basic religious education, including learning the Arabic
alphabet, Quranic recitation, and basic Islamic principles. Maktabs are typically small,
neighborhood-based institutions run by local teachers or imams. They are often
associated with mosques and serve as a fundamental stepping stone for children's
religious education. (Hence statement 1 is correct)
• Madrasas: Madrasas are Islamic educational institutions that offer a broader range of
subjects and a more advanced level of education compared to maktabs. They provide
instruction in subjects such as Quranic studies, Hadith (sayings and actions of Prophet
Muhammad), Islamic law, theology, Arabic language, and other religious sciences.
Madrasas can be found at various levels, ranging from primary to higher education.
Some madrasas are more specialized, focusing on specific areas of Islamic studies or
jurisprudence.
• The famous ‘Madrasas’ were the Muizzi, the Nasiri and the Firuzi madrasas in Delhi,
Mohammed Gawani’s madrasa in Bidar and Abul Fazl’s madrasa in Fatehpur Sikri.
(Hence statement 2 is incorrect)

Q.83) Solution (a)

Explanation:

• Learning was an intimate relationship between the teacher and the pupil, called the
Guru Shishya Parampara. The process of learning generally began with a religious
ceremony, and it was marked as the acceptance of a student by teacher. ‘Upanayana’,
which was a sacred, thread ceremony. (Hence statement 1 is correct)
• Charak Samhita was written by Charaka. It is an authentic and exhaustive work on
medicines. On the other hand, Sushruta worked in the field of surgery. His work is
known as Sushruta Samhita. (Hence statement 2 is incorrect)
• In the seventh and the eighth centuries, Ghatikas, or colleges attached to the temples
emerged as new centres of learning. The ‘Ghatikas’ provided Brahmanical education.
The medium of instruction was Sanskrit. (Hence statement 3 is incorrect)

Q.84) Solution (d)

Explanation:

Puranas

• The word 'puranas' literally means 'old'. They were the religious sectarian literature of
later times (around 4th-6th centuries) which had a well-defined perception of the past
.
IASBABA’S PRELIMS TEST SERIES (AIPTS) 2024 – TEST 7 (ART & CULTURE) SOLUTIONS

traditions. They reflect the Brahmanical social and religious values and also shed light
on the emergence and development of Hindu religious practices. The Puranas
generally discuss five topics in the time span of four ages/yugas (Satya, Treta, Dvapara
and Kali):
i. Sarga: describes why and how the universe was created.
ii. Pratisarga: is the creation of a new universe after the destruction of the old
one. Every creation ends in destruction, and every destruction is followed by a
new creation. The god of every new creation is Brahma.
iii. Vamsa: Genealogy of the sages, gods, and kings.
iv. Manvantara: It is a cosmic cyclic period of time identifying the duration or era,
reign, or age of a Manu, the progenitor of mankind.
v. Vamsanucharita: Text relating rise and fall of kings belonging to royal
dynasties, both the Suryavanshis who claimed their descent from the Sun and
Chandravanshis who claimed their descent from the Moon.(Hence statement
1 is correct)
• Traditionally, the Puranas are considered to be composed by Vyasa and it is commonly
accepted that the four yugas make up a mahayuga, that 1000 mahayugas make a kalpa,
that and every kalpa is further divided into 14 manvantaras which are presided over
by a specific Manu. Each Yuga is periodically destroyed and again, the recreation of
world occurs with the cyclical decline and revival of Dharma.
• It contains the stories of the kings, heroes, sages, and demigods, but it focuses on the
divine Hindu trimurti or trinity/three Gods: Brahma, Vishnu and Mahesh. (Hence
statement 2 is correct)
• The Puranas are further divided into 18 Mahapuranas (great Puranas such as Brahma,
Vishnu, Narada, Bhagavata, Padma, Garuda, Varaha, Matsya, Kurma, Linga, Shiva,
Skanda, Agni, Vamana, etc.) and numerous Upapuranas (secondary Puranas).
• Some Puranas such as the Brahma, Matsya, Harivamsha, Vishnu, Brahmanda and Vayu
provide useful information on historical dynasties (of Haryankas, Shishunagas, Nandas,
Mauryas and Shungas - till the Guptas). They also contain detailed accounts of
historical geography such as mountains, rivers, etc. (Hence statement 3 is correct)

Q.85) Solution (b)

Explanation:

• Rigveda: It is an ancient Indian collection of Vedic Sanskrit hymns. It is one of the four
canonical sacred texts of Hinduism known as the Vedas. The text is a collection of 1,028
hymns and 10,600 verses, organized into ten books (Mandalas). The Rigveda was
composed in the north western region of the Indian subcontinent, most likely between
1500 and 1200 BCE.
.
IASBABA’S PRELIMS TEST SERIES (AIPTS) 2024 – TEST 7 (ART & CULTURE) SOLUTIONS

• AtharvaVeda: It is the ‘knowledge storehouse of atharvas’ as, the procedures for


everyday life”. The text is the fourth Veda, but has been a late addition to the Vedic
scriptures of Hinduism. The Atharvaveda was likely compiled as about 1200 BCE - 1000
BCE.
• Vedanga: Six technical subjects related to the Vedas are traditionally known as
Vedanga “limbs of the Veda”. The Vedangas likely developed towards the end of the
Vedic period, The Vedangas likely developed towards the end of the Vedic period,
around or after the middle of the 1st millennium BCE.
• Natyasastra:The first formal mention of dance is found in Bharata's famous work
Natya Shastra, which is a most comprehensive and vivid treatise on the various facets
of Indian classical dance. The work was compiled probably between 200 BC and 200
AD and describes in detail the techniques, postures, emotions, ornaments, stage and
even the audience. Bharata Muni describes dance as the 'complete art' encompassing
within its scope all other forms of art- music, sculpture, poetry and drama.
• Rig Veda →Atharvav → Vedangas → Natyashashtra. (Hence option (b) is correct
answer)

Q.86) Solution (d)

Explanation:

• Namdev: He was a poet-saint from Maharashtra, who is significant to the Varkari sect
of Hinduism.He used the Marathi language to convey their beliefs rather than using
the traditional Sanskrit language that was essentially used by the Brahmin priests.
Namdev was influenced by Vaishnavism, and became widely known in India for his
devotional songs set to music (bhajankirtans). (Hence pair 1 is correct)
• Tulsidas: He was a realized soul and saint, poet, often called reformer and philosopher
from Ramanandi Sampradaya, in the lineage of Jagadguru Ramanandacharya
renowned for his devotion to the Lord Shri Rama. Tulsidas wrote several popular works
in Sanskrit and Awadhi; he is best known as the author of the epic Ramcharitmanas, a
retelling of the Sanskrit Ramayana based on Rama’s life in the vernacular Awadhi
dialect of Hindi. (Hence pair 2 is correct)
• Kabir: He was a 15th-century Indian mystic poet and saint, whose writings influenced
Hinduism’s Bhakti movement and his verses are found in Sikhism’s scripture Guru
Granth Sahib. His early life was in a Muslim family, but he was strongly influenced by
his teacher, the Hindu bhakti leader Ramananda. Kabir’s poems were in vernacular
Hindi, borrowing from various dialects including Avadhi, Braj, Bhojpuri. (Hence pair 2
is correct)
• Narsi Mehta: Gujarati He hailed from Gujarat where the Bhakti movement had
travelled to and then farther off to Kutch and Sind through Rajasthan and Punjab. He
was a Vaishnava devotee and sang the praises of Lord Krishna.He is especially revered
.
IASBABA’S PRELIMS TEST SERIES (AIPTS) 2024 – TEST 7 (ART & CULTURE) SOLUTIONS

in Gujarati literature, where he is acclaimed as its Adi Kavi. His bhajan Vaishnav Jan
was Mahatma Gandhi’s favourite and has become synonymous to him. (Hence Pair 4
is Incorrect)

Q.87) Solution (a)

Explanation:

Pali and Prakrit were the spoken languages of Indians after the Vedic period.Prakrit in the
widest sense of the term was indicative of any language that in any manner deviated from the
standard one i.e. Sanskrit. Pali is a combination of various dialects.These was adopted by
Buddhist and Jain sects in ancient India as their sacred languages.

• Pali:
• Lord Buddha (500 B.C.)Used Pali to give his sermons. This was also the
common mans’ language to reach the masses. All the Buddhist canonical
literature is in Pali which includes Tipitaka (threefold basket) the first basket,
Vinaya Pitaka, the second basket, Sutta Pitaka and the third basket, the
Abhidhamma Pitaka. (Hence statement 1 is correct)
• The Jataka Kathas are non-canonical Buddhist literature in which stories
relating to the former births of the Buddha (Bodhi-sattva or the would-be
Buddha) are narrated. These stories propagate Buddhist religious doctrines and
are available in both Sanskrit and Pali.
• Prakrit:
o Like the Buddhist stories, the Jain tales in general are didactic in character. They
are written in some forms of Prakrit. (Hence statement 2 is correct)
o Prakrit is well known for Gathasaptashati (700 verses) by Hala (300 A.D.), the
best example of erotic literature. It is a compilation of 700 verses along with
his own contribution of 44 poems. The characteristic of Prakrit poetry is its
subtlety; the inner meaning (Hiyaali) is its soul.
• Panchatantra:
o It is an ancient Indian work of political philosophy, in the form of a collection
of interrelated animal fables in Sanskrit verse and prose, arranged within a
frame story. (Hence statement 3 is incorrect)
o The surviving work is dated to about 300 BCE, but the fables are likely much
more ancient.
o It is likely a Hindu text and based on older oral traditions with “animal fables
that are as old as we are able to imagine”.

Q.88) Solution (d)


.
IASBABA’S PRELIMS TEST SERIES (AIPTS) 2024 – TEST 7 (ART & CULTURE) SOLUTIONS

Explanation:

Charvaka School or Lokayata Philosophy

• Brihaspati is considered the founder of this school, which is supposed to be one of the
earliest schools to develop a philosophical theory. The philosophy is old enough to find
mention in the Vedas and the Brihadarankya Upanishad. The Charvaka School was the
main propounder of the materialistic view to achieve salvation. As it was geared
towards the common people, the philosophy was soon dubbed as Lokayata or
something derived from the common people.
• The word 'Lokayata' also means a keen attachment to the physical and material world
(loka). The proponents argued for a complete disregard of any world beyond the world
that is inhabited by a person. (Hence statement 3 is correct)
• They denied the existence of any supernatural or divine agent who could regulate our
conduct on earth. They argued against the need to achieve salvation and also denied
the existence of brahm and God. They believed in anything that could be touched and
be experienced by the human senses. (Hence statement 1 is correct)

Some of their main teachings are the following:

o They argued against Gods and their representatives on the earth - the priestly
class. They argued that a Brahman manufactures false rituals so as to acquire
gifts (dakshina) from the followers.
o Man is the centre of all activities and he should enjoy himself as long as he
lives. He should consume all earthly goods and indulge in sensual pleasure.
o The Charvakas do not consider 'ether' as one of the five essential elements
because it cannot be experienced through perception. Hence, they say that the
Universe consists of only four elements: fire, earth, water and air.(Hence
statement 2 is correct)
o This school argues that there is no other world after this one, hence death is
the end of a human being and pleasure should be the ultimate objective of life.
Hence, they propounded the theory of 'eat, drink and be merry'.

Q.89) Solution (d)

Explanation:

Vaisheshika School

The Vaisheshika School believes in the physicality of the universe, offering a realistic and
objective philosophy that governs the universe. Kanada who wrote the basic text governing
the Vaisheshika philosophy is often considered the founder of this school. The proponents
argue that everything in the universe was created by the five main elements: fire, air, water,
.
IASBABA’S PRELIMS TEST SERIES (AIPTS) 2024 – TEST 7 (ART & CULTURE) SOLUTIONS

earth and ether (sky). These material elements are also called Dravya. They also argue that
reality has many categories, for example, action, attribute, genus, inherence, substance and
distinct quality.

The school has a very scientific approach and developed the atomictheory, i.e., all material
objects are made of atoms. Its proponents explain the phenomenon of this universe by
arguing that the atoms and molecules combined to make matter, which is the basis for
everything that can be physically touched or seen. This school was also responsible for the
beginning of physics in the Indian sub-continent. They are considered to be the propounders
of the mechanical process of the formation of this Universe.

• On the subject of God, even though the proponents support scientific thinking, they
believe in God and consider him the guiding force.
• They also believe that the laws of karma guide this universe, i.e., everything is based
on the actions of human beings. We are rewarded or punished according to our
actions.
• God decides the merits and the demerits of our actions and individuals are sent to
heaven or hell accordingly.
• They also believed in salvation, but it was parallel to the creation and destruction of
the Universe, which was a cyclic process and was decided by the wishes of God.

(Hence option (d) is correct answer)

Q.90) Solution (b)

Explanation:

The Archaeological Survey of India

The Archaeological Survey of India (ASI) is directly under the aegis of the Ministry of Culture,
and it is the foremost institution for archaeological research conducted across India. Its focus
is on the preservation of the physical and tangible heritage that has accumulated in India's
ancient monuments and archaeological sites. (Hence statement 1 is incorrect)

• The provisions of the Ancient Monuments and Archaeological Sites and Remains Act,
1958, guide the ASI.
• Another major legislation that directs the working of the ASI is the Antiquities and Art
Treasure Act, 1972. As per this Act, the ASI is to prevent the illegal export of Indian
antiquities.
• The ASI employs many trained archaeologists, architects, conservators, epigraphists
and others. It has other institutions under its aegis, such as museums, excavation
branches, epigraphy branches, building survey projects, a horticulture branch, and
temple survey projects.
.
IASBABA’S PRELIMS TEST SERIES (AIPTS) 2024 – TEST 7 (ART & CULTURE) SOLUTIONS

• One of the more specialised and one-of-a-kind divisions of the ASI are its Underwater
Archaeology Wing. The ASI also offers several diplomas and degrees for studying
archaeology and other specialised courses, such as epigraphy and museology, from the
Institute of Archaeology, New Delhi.

Indira Gandhi National Centre for the Arts

The Indira Gandhi National Centre for the Arts (IGNCA) was started in 1985. The IGNCA is an
autonomous institution that concentrates on research, conversion, display and dissemination
of all the arts. Although it concentrates on the Visual and Performing Arts, it also promotes
critical and creative literature. (Hence statement 2 is correct)

The IGNCA has nine functional units:

• Kala Nidhi – the multi-form library.


• Kala Kosa – devoted mainly to the study and publication of fundamental texts in Indian
languages.
• Janapada Sampada – engaged in lifestyle studies.
• Kaladarsana – the executive unit which transforms research and studies emanating
from the IGNCA into visible forms through exhibitions.
• Cultural Informatics – This applies technology tools for cultural preservation and
propagation.
• Adi Drishya Department – dedicated to the study of prehistoric rock art.
• Conservation Lab – pecializes in the areas of preventive conservation, conservation
training, conservation of books, manuscripts, paintings, and objects (metals, wooden
objects, ethnographic objects, etc).
• Sutradhara – the administrative section that acts as a spine supporting and
coordinating all the activities.
• Media Centre – Media Centre has been endeavouring to do audio/visual research
documentation and archiving them for prosperity and implementing worldwide
dissemination as well.

The main objective of the IGNCA is to be a major resource centre for written, oral and visual
art forms of India.

Centre for Cultural Resources and Training

The Ministry of Culture, Government of India, has instituted the Centre for Cutural Resources
and Training (CCRT) for linking education with culture. It was established in 1979 at the behest
of Dr. Kapila Vatsyayan and Smt. Kamala Devi Chattopadhyay. (Hence statement 3 is correct)

Although an autonomous body, the CCRT has been mandated by the government to
strengthen the foundation of the nation by making education, culture-based and meaningful.
.
IASBABA’S PRELIMS TEST SERIES (AIPTS) 2024 – TEST 7 (ART & CULTURE) SOLUTIONS

The CCRT has its headquarters in New Delhi and three regional centres at Udaipur in the west,
Hyderabad in the South and Guwahati in the North-East, to facilitate the widespread
dissemination of Indian art and culture.

CCRT not only focuses on students, but also raises understanding of the diversity of regional
cultures and languages in India among teachers, principals, and non-teaching/administrative
actors. This diversity must be reflected in the curriculum and reinforced via new and
innovative teaching approaches.

Some of these new methodologies suggested are:

• Organizing workshops to provide teachers with practical training and understanding in


crafts that may be taught as part of the school curriculum is one of the new techniques
offered.
• These activities might include everything from classical dance to music to a variety of
art forms that show off India's cultural richness.
• To develop an Indian art and culture curriculum for instructors, who would then teach
it to pupils.
• To establish a library to store scripts, digital images, audio and visual recordings, all of
which would be culturally oriented teaching aids focusing on rural India's arts and
crafts, as well as ways to revive and preserve them.
• The Government of India funds CCRT to grant scholarships to students, teachers, and
artists to aid them in their efforts to connect education and culture.
• Young Artists, Junior, and Senior Fellowships are awarded for more in-depth research.
• They also host the Cultural Heritage Young Leadership Program, which strives to
promote social ideals and community involvement among India's forward-thinking
youth.

Q.91) Solution (b)

Explanation:

Central Board of Film Certification

The government body that governs and directs censorship in India is called the Central Board
of Film Certification (CBFC). It was set up in 1950 under the name Central Board of Film
Censors, but the same was changed under the Act of 1952. It is directly under the directive of
the Ministry of Information and Broadcasting. Although the head office is in Mumbai, it has
many regional offices that deal specifically with the regional films. These offices are in Delhi,
Kolkata, Chennai, Bangalore, Guwahati, Cuttack, Thiruvananthapuram and Hyderabad. All of
these institutions provide the certificate to a film without which they cannot be screened in
the cinema theatres.
.
IASBABA’S PRELIMS TEST SERIES (AIPTS) 2024 – TEST 7 (ART & CULTURE) SOLUTIONS

• The CBFC is a statutory body and well-structured organisation and it has a Chairman
and governing members who are appointed by the government through the Ministry
of Information and Broadcasting. They can be appointed for a term of three years or
more, as per the government directive. The members are usually famous and talented
personalities from the film industry or other intellectuals. (Hence statement 1 is
correct)
• It is necessary for all films to get a Censor Board Certificate. Even foreign films that a
imported to India have to get a CBFC certification. (Hence statement 2 is incorrect)
• All the films that are dubbed from one language to another have to get a fresh
certificate to ensure that the language change is one not offensive in any manner. The
only exception to the CBFC certificate are films made especially for Doordarshan as
they are the official broadcaster for the Government of India and they have their own
set of rules for examining such films. CBFC certification is not required for television
programmes and serials. (Hence statement 3 is correct)
• In 2016, the Government of India had constituted the Shyam Benegal Committee to
lay down norms for film certification that take note of best practices in various parts
of the world and give sufficient and adequate space for artistic and creative expression.

The Committee submitted their recommendations and some of the major highlights of their
report are as follows:

• CBFC should only be a film certification body whose scope should be restricted to
categorising the suitability of the film to audience groups on the basis of age and
maturity.
• The committee has also made certain recommendations regarding the functioning of
the board and has stated that the Board, including Chairman, should only play the role
of a guiding mechanism for the CBFC, and not be involved in the day-to-day affairs of
certification of films.
• Online submission of applications as well as simplification of forms and accompanying
documentation.
• Recertification of a film for the purposes of telecast on television or for any other
purpose should be permitted.
• Regarding the categorisation of films, the Committee recommended that it should be
more specific and apart from U category, the UA category can be broken up into further
sub-categories - UA12+ and UA15+. The A category should also be sub-divided into A
and AC (Adult with Caution) categories.

Q.92) Solution (b)

Explanation:

Teesta Rangit Tourism festival:


.
IASBABA’S PRELIMS TEST SERIES (AIPTS) 2024 – TEST 7 (ART & CULTURE) SOLUTIONS

• The Teesta Rangit Tourism festival was held in separate phases in different places of
Darjeeling and Kalimpong districts of West Bengal.
• This festival is organized by Gorkhaland Territorial Administration (GTA) and Darjeeling
state tourism department and information and cultural affairs department.

(Hence option (b) is correct answer)

About Gorkhaland Territorial Administration (GTA):

o Purpose - administrative body for the Darjeeling and Kalimpong hills


o Headquarters - Darjeeling
o Chairman - Anit Thapa
• The Gorkhaland Territorial Administration (GTA) is an autonomous administrative body
in the Darjeeling Hills of West Bengal, India. It was formed through an agreement
reached between the Government of West Bengal and the Gorkha Janmukti Morcha
(GJM), a political party representing the interests of the Gorkha community.
• The GTA was established in 2012 as a result of the Gorkhaland movement, which
sought to create a separate state of Gorkhaland within India. While the demand for a
separate state of Gorkhaland has not been fulfilled, the GTA was seen as a measure to
address some of the aspirations of the Gorkha community and provide a degree of
self-governance.
• The GTA has administrative and executive powers over various subjects, including
education, health, tourism, agriculture, and others. It has its own legislative body
called the Gorkhaland Territorial Administration Sabha, which consists of elected
representatives from the Darjeeling Hills region.
• The GTA is headed by a Chief Executive who is appointed by the Governor of West
Bengal. The Chief Executive is supported by a Board of Administrators responsible for
the functioning of the GTA. The body operates within the framework of the
Constitution of India and functions under the supervision of the Government of West
Bengal.

Q.93) Solution (a)

Explanation:

Hornbill Festival

• The Hornbill Festival began in the Naga Heritage village of Kisama. The festival
coincides with the Statehood Day of Nagaland which is observed on December 1.
(Hence statement 1 is correct)
• It is tourism promotional extravaganza to revive, protect and preserve the richness and
uniqueness of the Naga heritage. It is also called as the ‘Festival of Festivals‘.
.
IASBABA’S PRELIMS TEST SERIES (AIPTS) 2024 – TEST 7 (ART & CULTURE) SOLUTIONS

• Hornbill Festival is celebrated in Nagaland every year in the first week of December. It
is one of the biggest indigenous festivals of the country. (Hence statement 2 is
incorrect)
• The festival is a tribute to Hornbill; the most admired and revered bird for the Nagas
for its qualities of alertness and grandeur.
• The majestic bird is closely identified with the social and cultural life of the Nagas as
reflected in tribal folklore, dances and songs.
• It is organized by the State Tourism and Art & Culture Departments. It is also supported
by the Union Government.
• The Hornbill Festival provides a colourful mixture of dances, performances, crafts,
parades, games, sports, food fairs and religious ceremonies.
• It was established on 1st December 1963 and was inaugurated by the then President
Dr. S Radhakrishnan.
• State bird of Nagaland is Blyth’s tragopan is a Schedule -I bird, according to Wildlife
(Protection) Act, 1972 and classified as Vulnerable (VU) by the IUCN. (Hence statement
3 is incorrect)

Q.94) Solution (b)

Explanation:

Aryabhatta:

• Aryabhatta was a fifth century mathematician, astronomer, astrologer and physicist.


He was a pioneer in the field of mathematics.
• At the age of 23, he wrote Aryabhattiya, which is a summary of mathematics of his
time. There are four sections in this scholarly work. In the first section he describes the
method of denoting big decimal numbers by alphabets. In the second section, we find
difficult questions from topics of modern-day Mathematics such as number theory,
geometry, trigonometry and Beejganita (algebra). The remaining two sections are on
astronomy.
• Aryabhatta showed that zero was not a numeral only but also a symbol and a concept.
Discovery of zero enabled Aryabhatta to find out the exact distance between the earth
and the moon. The discovery of zero also opened up a new dimension of negative
numerals.

Baudhayan:

• He was the first one ever to arrive at several concepts in Mathematics, which were
later rediscovered by the western world. The value of pi was first calculated by him.
• As you know, pi is useful in calculating the area and circumference of a circle. What is
known as Pythagoras theorem today is already found in Baudhayan’s Sulva Sutra,
.
IASBABA’S PRELIMS TEST SERIES (AIPTS) 2024 – TEST 7 (ART & CULTURE) SOLUTIONS

which was written several years before the age of Pythagoras. (Hence statement 1 is
incorrect)

Kanad:

• Kanad was a sixth century scientist of Vaisheshika School, one of the six systems of
Indian philosophy.
• His atomic theory can be a match to any modern atomic theory. (Hence statement 2
is correct)
• According to Kanad, material universe is made up of kanas, (anu/atom) which cannot
be seen through any human organ. These cannot be further subdivided. Thus, they are
indivisible and indestructible.

Charak:

• Charak is considered the father of ancient Indian science of medicine.


• He was the Raj Vaidya (royal doctor) in the court of Kanishka. (Hence statement 3 is
correct)
• His Charak Samhita is a remarkable book on medicine. It has the description of a large
number of diseases and gives methods of identifying their causes as well as the
method of their treatment.
• He was the first to talk about digestion, metabolism and immunity as important for
health and so medical science.
• In Charak Samhita, more stress has been laid on removing the cause of disease rather
than simply treating the illness. Charak also knew the fundamentals of Genetics.

Bhaskaracharya:

• Bhaskaracharya was the leading light of 12th Century. He was born at Bijapur,
Karnataka. He is famous for his book Siddanta Shiromani.
• It is divided into four sections: Lilavati (Arithmetic), Beejaganit (Algebra), Goladhyaya
(Sphere) and Grahaganit (mathematics of planets).
• Bhaskara introduced Chakrawat Method or the Cyclic Method to solve algebraic
equations. This method was rediscovered six centuries later by European
mathematicians, who called it inverse cycle. In the nineteenth century, an English man,
James Taylor, translated Lilavati and made this great work known to the world.

Brahamagupta:

• Brahmagupta (7th century AD) in his book Brahmasputa Siddhanta mentioned Zero as
a number.
• In his book, he also introduced negative numbers and described them as debts and
positive numbers as fortunes. (Hence statement 4 is incorrect)
• It also contained first clear description of Quadratic formula.
.
IASBABA’S PRELIMS TEST SERIES (AIPTS) 2024 – TEST 7 (ART & CULTURE) SOLUTIONS

Q.95) Solution (c)

Explanation:

Kumbh Mela

• The Kumbh Mela is the largest religious gathering in the world. (Hence statement 1 is
correct)
• Every day during the mela millions of people come to take a dip in the holy river. The
mela (gathering) is held on a rotational basis at four auspicious Hindu pilgrimage sites
- Prayagraj, Haridwar, Nashik-Trimbak and Ujjain
• According to Hindu mythology, during the 'Samudra Manthan', that is churning of the
ocean, ‘Amrit’ that is drink of immortality, was produced and stored in a 'Kumbh' (pot).
In the battle of the Devas and the Asuras, Lord Vishnu dropped drops of Amrit while
transporting the Kumbh. These places are the four sites where the Kumbh Mela is held.
• The mela is held at any given place after a time interval of 12 years. The exact dates
are determined according to the zodiac positions of the Sun, Moon and the planet
Jupiter. At Nashik and Ujjain, if the mela is held when a planet is in Leo (Simha in Hindu
astrology), it is called as Simhastha Kumbh.
• At Haridwar and Prayagraj, the Ardh-Kumbh Mela is held every sixth year. (Hence
statement 2 is correct)
• At this Kumbh Mela in 1915, All-India Hindu Sabha (later became Hindu Mahasabha)
was found.
• Places where the Kumbh Mela is held:

Place River
At the confluence of the Ganga, Yamuna and the mythical
Prayagraj (Uttar Pradesh)
Saraswati.
Haridwar (Uttarakhand) Ganga
Nashik-Trimbak
Godavari
(Maharashtra)
Ujjain (Madhya Pradesh) Shipra

• In 2017, Kumbh Mela was inscribed on the Representative List of Intangible Cultural
Heritage of Humanity by UNESCO.
• The Chinese traveler Xuanzang (Hiuen Tsang) describes a ritual organized by Emperor
Shiladitya (identified with Harsha) at the confluence of two rivers, in the kingdom of
Po-lo-ye-kia (identified with Prayaga). He also mentions that many hundreds took a
bath at the confluence, to wash away their sins. (Hence statement 3 is correct)
IASBABA’S PRELIMS TEST SERIES (AIPTS) 2024 – TEST 7 (ART & CULTURE) SOLUTIONS

Q.96) Solution (c)

Explanation:

Sair-e-Gul Faroshan:

• The festival is also known as 'Phool Walon Ki Sair' and is the annual three-day festival
of flowers conducted in Delhi. It is a symbol of communal harmony in which
procession of pankhas or palm leaf fans which are heavily decorated with flowers are
taken from the tomb of Khwaja Bakhtiyar Kaki in Mehrauli to the Yogmaya Temple.
• The festival was initially patronised by Mughal Emperor Akbar II (19th century). It was
banned by the British but reinstated in 1962 by J. L. Nehru.

Q.97) Solution (c)

Explanation:

Kheer Bhawani Festival

• Kheer Bhawani is a revered Hindu temple located in the Tula Mula village of Jammu
and Kashmir, India. The temple is dedicated to the goddess Ragnya Devi, who is
believed to be an incarnation of the Hindu goddess Durga. Kheer Bhawani is
considered a highly sacred site for Kashmiri Pandits, and the temple attracts devotees
from various parts of India.
• One of the unique aspects of Kheer Bhawani temple is its sacred spring, which is
believed to possess miraculous powers. The spring is surrounded by a large hexagonal
pool, and the water changes its color, which is said to signify the goddess's mood and
foretell future events. The devotees offer milk and kheer (a sweet rice pudding) to the
goddess as a form of devotion. It is believed that the goddess Ragnya Devi resides in
the spring and protects her devotees.
• The temple holds special significance during the annual festival of Kheer Bhawani, also
known as Mela Kheer Bhawani, which is celebrated in the months of May or June
according to the Hindu lunar calendar. Devotees from different parts of the country
visit the temple during this festival, seeking blessings and divine intervention. The
festival is marked by various rituals, prayers, and community gatherings.
• Kheer Bhawani is not only a place of religious importance but also serves as a symbol
of communal harmony and unity in the region. People from different faiths and
communities visit the temple, showcasing the syncretic traditions and cultural
diversity of Jammu and Kashmir.

(Hence option (c) is correct answer)


IASBABA’S PRELIMS TEST SERIES (AIPTS) 2024 – TEST 7 (ART & CULTURE) SOLUTIONS

Q.98) Solution (c)

Explanation:

India being an agriculture country, where 70% of population of India live in villages and
depend on agriculture. Therefore, most of the big event of fair and festivals are related to
cultivation.

Pongal:

• It is a four-day harvest festival celebrated by Tamilians around the world. (Hence


statement 1 is incorrect)
• It is celebrated in January and marks the beginning of Uttarayan that is six-month
northward journey of the sun.
• The word ‘Pongal’ means ‘to boil’ in Tamil, and the boiling of the first rice is on
important ritual performed during the festival.
• It is held in the month of ‘Thai’, a time when various crops such as rice sugar cane and
turmeric or harvested.
• It is an occasion for giving thanks to the Sun God and celebrating the life cycles that
give us grain.

Makar Sankranti:

• It is a harvest festival when farmers bring home their harvest.


• It marks the beginning of Sun’s journey from Southern to Northern Hemisphere.
(Hence statement 2 is correct)
• It is a festival day in the Hindu calendar, in reference to deity Surya (sun). It is observed
each year in January.
• It marks the first day of sun’s transit into the Makara (Capricorn), marking the end of
the month with the winter solstice and the start of longer days.

Ganga Sagar Mela:

• It is conducted in the month of January-February at the mouth of the river Hoogly


(southern tip of Sagar Island) West Bengal. (Hence statement 3 is correct)
• The Ganga Sagar Mela is primarily dedicated to the worship of Goddess Ganga. A holy
dip in the Ganges especially on the day of Makar sankranti is considered very
auspicious by the Hindus.
• Lakhs of pilgrims throng the site. The presence of Naga Sadhus lends a unique identity
to the fair.
• The famous Kapil Muni Temple is located on Sagar Island.
IASBABA’S PRELIMS TEST SERIES (AIPTS) 2024 – TEST 7 (ART & CULTURE) SOLUTIONS

Q.99) Solution (d)

Explanation:

• The study of coins is known as numismatics. It not only includes visual elements such
as Script and images on the coins but also metallurgical analysis. Ancient coins were
mostly minted in metals such as copper, silver, gold and lead.
• The earliest coins found in India contained certain symbols and were called punch
marked coins. They were made of silver and copper (sixth century BC onwards).
• The first coins to bear the names and images of rulers were issued by the Indo-Greeks,
who established control over the north-western part of the subcontinent (second
century BC).
• The Gupta Empire produced large numbers of gold coins depicting the Gupta kings
performing various rituals. This tradition of engraved coins continued till the arrival of
the Turkish Sultanate in North India. (Hence statement 1 is correct)
• Earliest Kushan coinage is generally attributed to Vima Kadphises. The Kushan coins
generally depicted iconographic forms drawn from Greek, Mesopotamian, Zorastrian
and Indian mythology.Siva, Buddha and Kartikeya were the major Indian deities
portrayed. Kushan gold coins influenced subsequent issues, notably those of the
Guptas. (Hence statement 2 is correct)
• Islamic calligraphy on coins was first introduced in India during the rule of the Delhi
Sultanate, specifically by the Sultanate's founder, Qutb-ud-din Aibak. Qutb-ud-din
Aibak, who was a slave general of the Ghurid Empire in Afghanistan, established the
Delhi Sultanate in 1206. He introduced Islamic coinage and incorporated Arabic
inscriptions and calligraphy on the coins. This marked a significant departure from the
previous Hindu and Buddhist coinage that did not typically feature such inscriptions.
The inclusion of Islamic calligraphy on coins became a common practice throughout
the subsequent dynasties of the Delhi Sultanate and the Mughal Empire, leaving a
lasting impact on Indian numismatics and Islamic art in the region. (Hence statement
3 is correct)

Q.100) Solution (c)

Explanation:

Jnanpith Award (instituted in1961)

• It is also known as the Gyanpeeth Award (Seat of knowledge) and is given for
outstanding literary achievement. It was instituted by the Bharitya Jnapith, trust run
by the Jain family famous for founding the newspaper The Times of India.
IASBABA’S PRELIMS TEST SERIES (AIPTS) 2024 – TEST 7 (ART & CULTURE) SOLUTIONS

• It is given back to Indian citizens who compose literature in one of the 22 languages
listed in Schedule VIII of the Indian constitution and English. Apart from the plaque
and a cash price of rupees 11 lakh, the winner is also given a bronze statue of Goddess
Saraswati. This award is not given posthumously.
• It is important to remember that there are 23 languages (22 + English) in which the
Jnanpith Award is given. (Hence statement 1 is incorrect)

The Tagore Award:

• The Tagore Award for Cultural Harmony is an international award established by the
Government of India in 2011. It is named after the renowned Indian poet, philosopher,
and Nobel laureate, Rabindranath Tagore. (Hence statement 2 is correct)
• The Tagore Award aims to recognize individuals, organizations, or institutions that
have made outstanding contributions to promoting cultural harmony and universal
values through their work in various fields such as art, music, dance, literature,
education, and social outstanding contributions to promoting cultural harmony and
universal values service.
• The award is presented annually by the President of India on or around May 7th,
which is the birth anniversary of Rabindranath Tagore. It carries a prize money of
rupees 1 crore Indian, a citation, and a plaque.

Saraswati Samman

• The Saraswati Samman is an annual award for outstanding prose or poetry literary
works in any of the 22 Indian languages listed in Schedule VIII of the Constitution of
India. (Hence statement 3 is incorrect)
• It is named after the Indian Goddess of learning and is considered to be among the
highest literary awards in India.
• The Saraswati Samman was instituted in 1991 by the K.K. Birla Foundation.
• It consists of rupees 15 lakh, a citation and a plaque. Candidates are selected from
among those whose literary works have been published in the past ten years.
• The first resipient was Harivansh Rai Bachchan.

You might also like